SlideShare a Scribd company logo
1 of 114
 Main secretory hormone T4; T3 more potent
 Free hormone only active 1.FT4 2.FT3(#2)
 TSH is the best initial test for diagnosis(#1)
 TBG (#3)( increase=endogenous T4)
 Antimicrosomal antibody
 Radioactive iodine uptake scan(RAIS)-check
functional status of thyroid
 Total T3,T4 are not always accurate:
1. Increase TBG level:
- Pregnancy
- OCP
2. Decrease TBG level:
-nephrotic syndrome
-liver ds
 An 86-year-old female is brought to the emergency
department from her nursing home facility. She has
been more lethargic and less communicative over the
past 3 days. The patient's only complaint is generalized
weakness. Her past medical history is significant for
mild dementia, well-controlled hypertension, and
hyperlipidemia. Her medications include
hydrochlorothiazide, losartan, and simvastatin. Triage
vitals are T 99.0F, HR 104; RR 20; BP 120/86mmHg.
EKG is shown in Figure A. Blood counts and metabolic
panel are within normal limits. In addition to an
infection work-up, which of the following would be the
most useful to include in the evaluation of this patient?
 1. Creatinine kinase
 2. Calcium, magnesium, phosphorus levels
 3. Liver function tests
 4. Thyroid stimulating hormone
 5. Lead level
 PREFERRED RESPONSE ▼ 4
 In an elderly patient presenting with new-onset atrial fibrillation (EKG in Figure A) and
hypoactive altered mental status, suspect apathetic thyrotoxicosis, an atypical presentation of
hyperthyroidism. A thyroid simulating hormone level (TSH) is the most appropriate screening
test for hyperthyroidism.
 The diagnosis of hyperthyroidism in the elderly patient requires a high index of suspicion, as the
disease does not always present with the classic symptoms of hyperthyroidism, such as heat
intolerance, weight loss, and hyperactivity. In elderly patients, the most common presenting
symptoms are cardiac, specifically atrial fibrillation, congestive heart failure, and angina.
Furthermore, these symptoms can be confused with primary cardiac pathology or masked by
home medications (i.e. beta-blockers). Apathetic thyrotoxicosis manifests with apathy,
weakness, depression, and lethargy.
 In their review, Espino et al. discuss the work-up of altered mental status in the elderly patient.
They recommend first distinguishing delirium from dementia or depression based on the
patient's history and exam. A Mini-Mental State Exam may be helpful in determining the degree
of impairment (Illustration A). Treatment is based on the underlying cause.
 In a case series of three patients presenting with apathetic thyrotoxicosis, Arnold et al. report
presenting symptoms of anorexia, gradual weight loss, restlessness, lethargy, weakness,
dyspnea on exertion, depression, irritability, forgetfulness, anhedonia, CHF, dry skin, pedal
edema, and polymyalgia. The three patients in this case series were female age 59-62 years
old.
 Figure A shows an EKG of a patient in atrial fibrillation.
 Illustration A shows a Mini-Mental State Examination. The test is comprised of 30 points
assessing for attention, memory, language, orientation, and registration. A score below 24 is
considered abnormal.
 Incorrect Answers:
 Answer 1: A creatinine kinase level may be useful in a
patient who was found down for a prolonged or
undetermined amount of time.
 Answer 2: Although hypocalcemia can result in altered
mental status, in a patient with new onset atrial
fibrillation without complaint of abdominal pain,
tetany, or muscle cramps, hyperthyroidism should be
ruled out first.
 Answer 3: Liver function tests (LFTs) should be
obtained in a patient with known or suspected liver
disease where hepatic encephalopathy is suspected.
Hepatic encephalopathy presents with new-onset
dementia, seizures, coma, and asterixis.
 Answer 5: A lead level should be obtained in a patient
presenting with anemia, abdominal colic, and
encephalopathy.
1
2
3
 Graves ds(MC)
 Toxic multinodular goiter
 Subacute thyroiditis
 Extrathyroid source
-thyrotoxicosis factitious
-stroma ovarii
 Excess TSH (rare)
 Hyperthyroidism + diffuse goiter +
exophthalmus + Dermatopathy
 Women(4th decade)
 Autoantibody (Thyroid stimulating Ig)
 Ass with other Autoimmune disorders:
-pernicious anemia
-myasthenia
-diabetes
 Most common cause of hyperthyroidism
 An autoimmune disease with stimulating anti-TSH receptor
antibodies
 a type II hypersensitivity
 anti-microsomal, anti-thyroglobulin antibodies also present
 Female dominant HLA-B8, Dr3 association
 Often incited during stress e.g. childbirth, infection, steroid
withdrawal
Physical exam
 symmetrical, non-tender thyroid enlargement
 ophthalmopathy (proptosis, exophthalmos) due to
glycosaminoglycan deposition
 pretibial myxedema
 digital swelling
Serology
 ↑ total serum T4
 ↑ free T4
 ↓ serum TSH
 diffusely ↑ 123I uptake
Treatment
Medical
 β-blockers
 thionamides
 result in reduced hormone synthesis
 PTU and methimazole
 131I ablation
 hypothyroidism may result
 May cause transient worsening of exophtalmos or
hyperthyroid symptoms due to release of thyroid hormone with
thyroid cell destruction
 Prevention: pretreatment with glucocorticoids
Prognosis, Prevention, and Comlications
 Stress-induced catecholamine surge may be fatal by
arrhythmia
 Pregnancy complications anti-TSH receptor antibodies may
cross placenta and produce hyperthryoidism in the fetus
 Nervous system(yonger)
 Inability to sleep,tremor
 Diarrhea,sweat
 Heat intolerance
 Weight loss(despite increase appetite)
 Dypnea,palpitation,angina,cardiac
failure,myopathy(old pt)
 Pretibial myxedema
 Palmar erythema
 Menstrual irregularity
 Osteoporosis,hypercalcemia
 Thyroid studies(#1)
-suppress TSH
-serum FT3,4 elevated
 RAI uptake(diffuse uptake) dy/dx solitary
uptake=toxic nodular goiter
 Thyroid stimulating Ig (TSI) (usually not
needed)
Treatment of Acute Hyperthyroidism and "Thyroid Storm"
1. Propranolol: blocks target organ effect, inhibits
peripheral conversion of T4T3
2. Thiourea drugs (methimazole and propylthiouracil) :
blocks hormone production(PTU safer in pregnancy)
3. Iodinated contrast material (iopanoic acid and ipodate):
blocks the peripheral conversion of T4 to the more active
T3; also blocks the release of existing hormone
4. Steroids (hydrocortisone)
5. Radioactive iodine: ablates the gland for a permanent
cure
 Thiourea drug may cause
neutropenia(agranulocytosis)stop and
change medication immediately
 Long term treatment:
-RAI ablationfollow them twice a year
prevent them getting hypothyroidism
-Thyroidectomy (rarely)-->reserve and use in
pregnancy because we cant use RAI ablation in
pregnancy
 PregnancyMethimazole is contraindicated.
 Steroids are the best initial therapy.
Radiation is used in those not responding to
steroids. Severe cases may need
decompressive surgery
 Non-autoimmune
 Consequences of simple goiter
 Nodular goiter on examination
 Disease of the elderly
 Presentation
-Arrthymia, CHF(cardiac manifestation)
 No opthalmopathy
 Treatment: radioactive iodine
 Extreme form of thyrotoxicosis
 Precipitated by stress,surgery or trauma
 Manifestation:
-High fever + cardiac manifestation
-hypotension,CHF
-delirium,coma
 Primary(95%)
1. Chronic thyroiditis(hashimato)
2. Post-ablative surgery,RAI
3. Drugs: lithium,amiodarone
4. Heritable biosynthethic defect(enzymatic
deficiency)(rare)
5. Iodine deficiency
 Suprathyroid cause: pituitary induces
(secondary) or hypothalamic
induced(tertiary)
 Acromegaly
 Bilateral carpal tunnel syndrome
 Cold intolerance
 Pseudodementia
 Increase weight
 Dry hair and skin
 Hoarse voice
 Slow deep tendion reflex
 Elevated cholesterol
 Cretinism(in newborn) and juvenile
hypothyroidism.
 Persistent physiology jaundice,hoarse
cry,constipation,somnolence and feeding problem.
 In later months,delayed mile-stones and dwarfism,
coarse features, protruding tongue, broad flat
nose, widely set eyes, sparse hair, dry skin
,protuberant abdomen, potbelly with umbilical
hernia, impaired mental development, retarded
bone age, and delayed dentition.
 Restore metabolic state
 Levothyroxine (T4) administered with
monitoring of TSH/T3 level (it takes 6 weeks
to stabilize after dosing changes).
 If there is a strong suspicion of suprethyroid
hypothyroidism/hypothalamic /pituitary
origin given hydrocortisone first then replace
thyroid hormone
Older age presenting as:
 Mental confusion
 Hypothermia
 Bradycardia
 Hyponatremia
 Hypoglycemia
 Hypercapnia
 Leukocytopenia
 Decrease hematocrit
 increaseCPK
 Decrease EKG voltage
Define:Inflammation of thyroid
 Subacute thyroiditis
 Hashimato thyroiditis
 Lymphocytic thyroiditis
 Reidel thyroiditis
 Granulomatous,giant cell,or the Quervan thyroiditis
 Viral origin and follows upper respiratory infection
symptoms include:
-malaise,fever,pain over thyroid,and pain refer to lower
jaw,ears,neck and arms.
 Thyroid gland is enlarged and firm
 Lab test:
1.Increase ESR
2.Initial increase T3 and T4 follow by hyopthyroidism
3. decrease RAI uptake
 Rx: symptomatic NSAIDs,prednisone and propanolol.
 May smolder for months but eventually wil lreturn to normal
function
 A 34-year-old woman presents with a pain in
the neck when with a mass. These symptoms
started following a viral upper respiratory
infection she had last week. On exam her
thyroid is enlarged and tender to palpation. A
biopsy is shown. (subacute granulomatous
thyroiditis)
27year old woman comes to your office for
“throat pain” for 1week and palpitation. She
had no prior hx . Not taking any medication.
BP 142/92,PP:112.
Oropharynx normal ; thyroid painful and
diffusely enlarged.
TSH < 0.01
T4 : 14.4 (increase)
RAI uptake scan 3%(low)
What would be the next most appropriate
test? Treatment?
 Subacute Thyroiditis.
 Check ESR
 Treatment : only symptomatic treatment
(pain)
32yo man admitted to ICU with delirium is
treated with presumed sepsis.
Thyroid hormone drawn on the day of
admission show a very low FT3. The TSH is
normal.
There is concern about myxedema coma given
that he had 2 history of “thyroid nodule”.
What is the most likely diagnosis.
 Apparent alterations thyroid hormone level with normal thyroid gland funciton
 peripheral activity of outer ring deiodinase is ↓ normally converts T4 to T3
 peripheral activity of inner ring deiodinase is ↑ normally converts T4 to reverse T3
 Associated with
 cancer
 cardiac disease (MI, CHF)
 CRF
 sepsis
Serology
 ↓ total serum T4/T3
 ↓ serum TSH
 ↑ reverse T3
 Management
 Treat and manage primary underlying illness
 Likely no need to start thyroid hormone replacement (controversial)
 Recheck thyroid function studies after illness has resolved.
 A 73-year-old female with a history of COPD and
hypertension presents to the emergency room
with a primary complaint of altered mental
status. Her husband reports that the patient was
suffering from worsening fatigue and a productive
cough over the last several days. Chest
radiographs in the emergency department suggest
a diagnosis of pneumonia. Blood cultures are
obtained and ultimately grow S. pneumoniae. The
patient is admitted to the ICU for management of
pneumonia and sepsis. During the course of her
admission, thyroid function studies are ordered
and show decreased levels of total serum T3 and
T4, high-normal TSH, and increased levels of
reverse T3. Which of the following is the most
appropriate next step in management of this
patient's abnormal thyroid function test results?
A. Initiate thyroxine
B. Radioactive iodine thyroid uptake test
C. Thyroid ultrasound
D. Fine needle aspiration thyroid biopsy
E. Continue to monitor patient and recheck
thyroid function after illness resolves
 This patient with pneumonia and sepsis as well as decreased levels of thyroid
hormones is suffering from euthyroid sick syndrome. Management of euthyroid sick
syndrome entails treatment of the primary underlying disease and rechecking thyroid
function after resolution of the illness.
 Euthyroid sick syndrome is characterized by abnormal thyroid function tests
concurrent with non-thyroid illness. Specifically, there are increased levels of reverse
T3 and decreased levels of total T3. The effect on total T4 and TSH is more variable
and less predictable. Euthyroid sick syndrome may present in the setting of
underlying malignancy, cardiac disease such as myocardial infarction or congestive
heart failure, renal failure, or sepsis.
 Pittman reviews the evaluation and management of patients with abnormal thyroid
function tests. Patients who are critically ill may have a wide array of abnormal
thyroid function tests secondary to sick euthyroid syndrome. For the majority of
patients, these abnormal results are due to the underlying disease and NOT a primary
thyroid disorder. Management includes, first and foremost, addressing the underlying
illness, with repeat thyroid function testing after recovery.
 McIver and Gorman review euthyroid sick syndrome. The thyroid hormone
abnormalities seen are likely due to cytokines or other inflammatory mediators acting
on the hypothalamus, pituitary, and/or thyroid gland. The degree of thyroid hormone
disturbance has been shown to correlate with the severity of illness; lower levels of
thyroid hormones may predict a poorer prognosis. The use of thyroid hormone
replacement therapy in euthyroid sick syndrome remains controversial.
 Answer A: There are no prospective studies
available that demonstrate the efficacy or utility
of thyroid replacement therapy for euthyroid
sick syndrome. Thyroid function abnormalities
are typically reversible with treatment and
resolution of the underlying causative illness.
 Answers B,C: Imaging studies such as thyroid
ultrasound or radioactive thyroid uptake test are
not necessary in euthyroid sick syndrome.
 Answer D: Thyroid biopsy is utilized to rule out
thyroid cancer and diagnose thyroid nodules
identified on imaging studies; it has no utility in
euthyroid sick syndrome.
 A 27-year-old female presents to the emergency
room with palpitations. Vitals are stable with the
exception of tachycardia. On physical examination,
she appears gaunt but has no goiter or proptosis.
Serum thyroglobulin and TSH are low. T3 is elevated.
A radioactive iodine study is performed which shows
low uptake in the thyroid gland. What is the most
likely diagnosis? Topic Review Topic
 1. Graves' disease
 2. Toxic multinodular goiter
 3. Factitious thyrotoxicosis
 4. Papillary thyroid carcinoma
 5. Iatrogenic hypothyroidism
 PREFERRED RESPONSE ▼ 3
 This patient has signs and symptoms of hyperthyroidism.
Low uptake of radioactive iodine in conjunction with a low
TSH and thyroglobulin are characteristic of excessive
doses of levothyroxine (or factitious thyrotoxicosis).
 Patients with factitious thyrotoxicosis have signs and
symptoms of thyrotoxicosis without goiter or
exophthalmos. Consider this diagnosis in a patient who has
access to levothyroxine such as a nurse or a pharmacist.
Recall the other common causes of hyperthyroidism:
Graves' disease (diffuse toxic goiter), Plummer's disease
(multinodular toxic goiter), toxic thyroid adenoma,
Hashimoto's thyroiditis (which may cause transient
hyperthyroidism before causing hypothyroidism),
postpartum thyroiditis, and iodine induced
hyperthyroidism. All these patients may present with the
classic clinical features including nervousness, hand
tremor, sweating, weight loss, diarrhea, and palpitations.
Treatment may be pharmacologic via thionamides such as
PTU and methimazole, or may consist of radioactive
iodine 131, and even surgical subtotal thyroidectomy.
 incorrect Answers:
 Answer 1: In Graves' disease, the patient would have
a goiter and the radioiodide scan would show diffuse
uptake.
 Answer 2: In toxic multinodular goiter (Plummer's
disease), there would be patchy uptake on the
thyroid scan. This form of hyperthyroidism is also
more common in elderly patients.
 Answer 4: Thyroid carcinomas are usually non-
functioning nodules.
 Answer 5: This patient's symptoms are indicative of
hyperthyroidism, not hypothyroidism. Iatrogenic
hypothyroidism occurs after radioiodine therapy,
thyroidectomy, or may occur with certain
medications such as Lithium.
 A 32-year-old woman presents to your office with a one
month history of heat intolerance, racing heart,
unintentional 4 pound weight loss, and sweating. On
physical examination there is a non-tender enlarged
thyroid without evidence of proptosis or exophthalmos.
Lab tests return with increased total T4, decreased TSH,
and increased free T4. A radioactive iodine uptake exam
is ordered and the imaging is displayed in Figure A. What
is the most appropriate definitive treatment option for
this patient? Topic Review Topic
 FIGURES: A
 1. Watchful waiting for symptom progression or
recession
 2. Beta-blockade alone
 3. Anti-thyroid medications alone
 4. Radioactive iodine ablation alone
 5. Radioactive iodine ablation and prophylactic
glucocorticoids
 PREFERRED RESPONSE ▼ 4
 DISCUSSION: The patient in the above vignette is experiencing symptoms associated
with Graves' disease. In the US, the definitive treatment for Graves' with mild or no
ophthalmopathy is radioactive iodine ablation.
 Graves' disease is an autoimmune disease in which thyroid stimulating
immunoglobulins bind to TSH-receptors. There tends to be female predominance,
and symptoms are often precipitated by stress (e.g. childbirth, infection). Graves'
often presents with heat intolerance, tachycardia, sweating, and a non-tender
thyroid enlargement. Some characteristic physical exam findings are demonstrated
in Illustrations A-C. Radioactive iodine uptake scans will show diffuse uptake.
Indications for radioactive iodine over antithyroid agents include a large thyroid
gland, multiple symptoms of thyrotoxicosis, high levels of thyroxine, and high titers
of TSI. Additionally, anti-thyroid medication use alone has been shown to result in a
higher rate of relapse when compared with radioactive iodine ablation.
 Reid et al. explore hyperthyroidism causes and treatment options. Graves' disease
is the most common cause of hyperthyroidism. Graves' can be treated with
radioactive iodine, anti-thyroid medications, or surgery, but in the US, radioactive
iodine ablation is the treatment of choice in patients without contraindications. Of
note, an infiltrative ophthalmopathy is noted in approximately 50% of these
patients.
 Marinò et al. report on the genetic and non-genetic factors of Graves'. Some of the
environmental factors that have been shown to be associated with this pathology
are female gender, periods of immune reconstitution, immune modulation, iodine,
smoking, and physiologic stress. The major genes that have been found to
predispose patients to Graves' disease are: HLA complex, CD40, CTLA-4, PTPN22,
FCRL3, Thyroglobulin, & TSH-R. With further study, the goal is to establish a causal
treatment pattern based on the defining etiology and genetics involved.
 Incorrect Answers:
 Answer 1: Watchful waiting is not an appropriate
treatment option for an individual with symptomatic
Graves' disease.
 Answer 2: Beta-blockade alone would help with some of
the hyper-sympathetic symptoms of Graves' disease
(e.g. tachycardia), but is not considered a definitive
treatment.
 Answer 3: In the US, anti-thyroid mediations alone
would not be considered an appropriate treatment
option for a patient with Graves' disease and associated
symptomatic tachycardia that could be treated with
beta-blockers as well. When methimazole or
propylthiouracil is used alone, there is an increased risk
of relapse of hyperthyroidism at a later time.
 Answer 5: Radioactive iodine ablation with prophylactic
glucocorticoids would be an appropriate treatment
option for someone with moderate to severe
exophthalmos as thyroid destruction during ablation
can cause worsening of symptoms.
 PREFERRED RESPONSE ▼ 4
 DISCUSSION: The therapy for Graves' disease with the best long-
term outcome is radioactive iodine therapy. Contraindications are
pregnancy and severe ophthalmopathy.
 Graves' disease (also known as diffuse toxic goiter) is the most
common cause of hyperthyroidism, responsible for approximately
80% of all cases. It is an autoimmune disorder caused by the
production of thyroid-stimulating antibody that bind to the TSH
receptors on thyroid cells causing hormone synthesis. Treatment
may include pharmacological therapy with propranolol for
immediate control of symptoms as well as methimazole and
propyltiouracil (PTU). However, the most successful therapy in the
long term is radioactive iodine ablation therapy. This treatment is
contraindicated in pregnant women in which PTU is preferred.
 Recall the other common causes of hyperthyroidism: Plummer's
disease, toxic thyroid adenoma, Hashimoto's thyroiditis,
postpartum thyroiditis, iatrogenic hyperthyroidism, and iodine
induced hyperthyoridism. All these patients may present with the
classic clinical features including nervousness, hand tremor,
sweating, weight loss, diarrhea, and palpitations. Treatment may
be pharmacologic via thionamides such as PTU and methimazole,
with radioactive iodine 131, or via surgical subtotal
thyroidectomy.
 Incorrect Answers:
 Answer 1: Levothyroxine is the treatment for
hypothyroidism but would make hyperthyroidism
worse.
 Answer 2: PTU is the therapeutic choice for a
pregnant woman with Graves' but is inferior to
radioactive iodine in the long term.
 Answer 3: Methimazole is a pharmacologic option
but it is not as effective as radioactive iodine in
the long term. Notably, it is contraindicated in
pregnancy.
 Answer 5: Propranolol is good for immediate
control of adrenergic symptoms but is not as
efficacious as radioactive iodine in the long
term.
 A 45-year-old female presents to her primary
care clinic with symptoms of palpitations,
hyperhidrosis, tremor and general hyperactivity.
A radioactive iodine update scan of this patient
showing decreased thyroid uptake would
effectively rule out which condition? Topic
Review Topic
 1. Subacute painless thyroiditis
 2. Subacute granulomatous thyroiditis
 3. Graves' disease
 4. Iodine-induced thyroid toxicosis
 5. Levothyroxineoverdose
 REFERRED RESPONSE ▼ 3
 DISCUSSION: The patient in this vignette most likely
has hyperthyroidism, as demonstrated by her
symptoms of hypermetabolism. It is important to
distinguish high and low uptake forms of
hyperthyroidism, of which the only high uptake-form
listed is Graves disease.
 There are several types of thyroiditis, one form of
which is subacute granulomatous thyroiditis. This
condition can present initially with either hypo or
hyperthyroidism. In the case of initial hyperthyroidism,
the destruction of the thyroid gland causes a release of
previously synthesized thyroid hormone but will
ultimately convert to a hypothyroid state as these
stores are consumed but not replaced. It is important
to note that the initial hyperthyroidism may progress
to thyroid storm, which may be fatal. Subacute
granulomatous thyroiditis can also initially present as a
hypothyroid state usually following a viral illness, and
in these cases is usually self-limited.
 ) A 61-year-old man presents to the emergency
room complaining a racing heart, sweats, and
diarrhea for 2 weeks. Review of systems is positive
for unintentional weight loss of 10 pounds in 1
month. Serum TSH is found to be 0.02 mIU/L
(normal 0.5 - 5.0 mIU/L). The patient is shown in
Figure A. If the patient is treated with I-131
radioiodine therapy, which of the following is the
most likely complication?
 FIGURES: A
 1. Agranulocytosis
 2. Increased total cancer mortality
 3. Hyperthyroidism
 4. Hypothyroidism
 5. Hypoparathyroidism
 PREFERRED RESPONSE ▼ 4
 DISCUSSION: The clinical presentation is
consistent with Graves' disease. Hypothyroidism
(due to radiation thyroiditis) may occur in the
treatment of Graves' disease with I-131
radioiodine therapy.
 In Graves' disease, the entire thyroid gland is
hyperfunctional. Uptake of the radioactive
isotope of iodine throughout the gland results in
effective ablation but may destroy too much
thyroid tissue, resulting in a hypothyroid state.
More than 75% of patients become hypothyroid
following radioactive iodine thyroid ablation.
 Incorrect Answers:
Answer 1: Agranulocytosis is the most feared
complication of the anti-thyroid drugs
propylthiouracil and methimazole.
Answer 2: Ron et al., in a study of over 35,000
patients, found no increased risk of total cancer
mortality following I-131 treatment. There may be a
slightly elevated risk of thyroid cancer mortality
following I-131 treatment, but underlying thyroid
disease appears to play a role.
Answer 3: While possible if insufficient dose of I-131
is given, hypothyroidism is much more common.
Answer 5: Hypoparathyroidism occurs if the
parathyroid glands are incidentally removed during
total thyroidectomy.
 A 69-year-old man presents to the general medical clinic
with palpitations. He has a history of an endocrine
disorder that he reports has caused him to have chronic
diarrhea and weight loss. His vital signs are temperature
37 degrees Celsius, blood pressure 130/85, heart rate
141, and respiratory rate of 18 with an oxygen saturation
of 99% on room air. His pulse is irregular on physical
examination. He is mentating normally and is in no acute
distress. His exam is also notable for hyperreflexia and
enlargement around his neck. An EKG reveals the
following in figure A. What would be the next best step
in management of this patient's chief complaint? Topic
Review Topic
 FIGURES: A
 1. Emergent cardioversion
 2. Administration of propranolol
 3. Administration of amiodarone
 4. Iodine 131 ablation
 5. Administration of methimazole
 PREFERRED RESPONSE ▼ 2
 DISCUSSION: In a patient with atrial fibrillation or
tachycardia secondary to hyperthyroidism, the initial
appropriate treatment is a beta blocker if that patient is
stable.
 Atrial fibrillation occurs when conduction in the atria fires
in a continuous chaotic pattern resulting in an irregular,
rapid ventricular rate. There are multiple causes including
coronary artery disease, myocardial infarction,
hypertension, mitral valve disease, pericarditis, pulmonary
disease, alcohol intake, and stress. Another cause is
hyperthyroidism. Clinically patients present with
palpitations, dizziness, angina, and an irregular pulse. On
EKG, the clinician should look for irregular RR intervals and
tachycardia. Treatment depends on whether the patient is
stable or unstable. In the unstable patient, immediate
electrical cardioversion to sinus rhythm is indicated. In a
stable patient, treatment focuses on rate control (target 60-
100) with beta blockers or calcium channel blockers. After
rate is controlled, rhythm can be addressed with
cardioversion.
 Image A depicts the classic irregularly irregular
tachycardia indicative of atrial fibrillation. Note the
absence of P waves.
 Incorrect Answers:
 Answer 1: Emergent cardioversion is indicated in
management of atrial fibrillation when the patient is
not stable. However, this patient is stable and could be
treated with a beta blocker.
 Answer 3: Administration of amiodarone is used in the
treatment of post myocardial infarction arrhythmias
such as ventricular tachycardia but would not be
indicated in this case.
 Answers 4 and 5: These are potential options in the
management of Grave's hyperthyroidism but would not
immediately address the patient's chief complaint
which is palpitations secondary to atrial fibrillation and
tachycardia.
 A 36-year-old G1P0 female presents to labor and
delivery in the 38th week of her pregnancy and
undergoes an uncomplicated spontaneous vaginal
delivery. Shortly after birth, the child is noted to
have dysphagia, irritability, frequent stooling, and
increased appetite. The mother notes no history of
drug or medication use during the pregnancy. The
newborn's CBC is within normal limits. Thyroid
studies reveal an increased free T4 in the newborn.
Which of the following is the most likely the cause
of this infant's presentation?
 1. Initial presentation of DiGeorge syndrome
 2. Intrauterine toxoplasmosis infection
 3. Maternal iodine deficiency
 4. Maternal history of Graves' disease treated with
radioactive thyroid ablation 10 years ago
 5. Maternal history of Hashimoto's thyroiditis
 PREFERRED RESPONSE ▼ 4
 DISCUSSION: Pregnant mothers with Graves' disease, even after being
treated surgically with thyroidectomy, can have persistent levels of
thyroid stimulating immunoglobulin that can cross the placenta and cause
thyrotoxicosis in the newborn, as seen in this vignette.
 Graves' disease is the most common cause of hyperthyroidism. This
condition is an autoimmune disorder with stimulating anti-TSH receptor
antibodies (type II hypersensitivity). Anti-microsomal and anti-
thyroglobulin antibodies are also present. Graves' disease is a female
dominant disorder and has associations with HLA-B8 and DR3.
Thyrotoxicosis is most often incited during stress (e.g. childbirth,
infection, or steroid withdrawal). Fetal hyperthyroidism can present with
low birth weight, microcephaly, warm wet skin, dysphagia, irritability,
increased appetite with poor weight gain, exophthalmos, and diffuse
goiter.
 Bishnoi and Sachmechi discuss hyperthyroid disease management during
pregnancy. Thyroid-stimulating hormone (TSH) is induced in normal
pregnancy; however, in patients who are already hyperthyroid this may
result in thyrotoxicosis leading to abortion, stillbirth, neonatal death, and
low birth weight. They report that the main cause of thyrotoxicosis in
pregnancy is Graves' disease, which may be treated with antithyroid drugs
or surgery. Importantly, many thyroid conditions and treatments may
directly affect the fetus, such as autoimmune postpartum thyroiditis.
 Incorrect Answers:
 Answer 1: Hypoparathyroidism, not
hyperthyroidism, is seen in DiGeorge syndrome.
 Answer 2: Intrauterine toxoplasmosis infection is
not associated with hyperthyroidism.
 Answer 3: Maternal iodine deficiency is
associated with hypothyroidism, not
hyperthyroidism. Congenital hypothyroidism
presents with poor feeding, lethargy, hypotonia,
coarse facial features, large protruding tongue,
and developmental delay at 6-12 weeks of life.
 Answer 5: Maternal history of Hashimoto's
thyroiditis is not associated with neonatal
hyperthyroidism.
 A 32-year-old woman with Graves' disease is
undergoing treatment with radioactive iodine. Her
initial presentation consisted of symptoms of sweating,
weight-loss, and intermittent palpitations along with a
physical examination significant for mild-to-moderate
exophthalmos. After completing one week of
radioactive iodine therapy, she reports worsening of her
proptosis, with increased pain and worsened periorbital
edema. Which of the following could have prevented
the worsening of this patient's exophthalmos? Topic
Review Topic
 1. Giving a larger dose of radioiodine therapy
 2. Initiation of beta-blocker at time of radioiodine
therapy
 3. Begin methimazole concurrent with initiating
radioiodine therapy
 4. Pre-treatment with prednisone prior to initiating
radioiodine therapy
 5. This is an expected outcome from radioactive iodine
therapy, no preventive options are availab
 PREFERRED RESPONSE ▼ 4
 DISCUSSION: This patient's Graves' ophthalmopathy was
worsened by the initiation of radioactive iodine.
Pretreatment with glucocorticoids, such as prednisone,
may prevent this adverse effect.
 Worsening of exophthalmos is due to the release of
excess thyroid hormone during the destruction of
thyroid cells by the radioactive iodine. Administration
of radioactive iodine may also precipitate a thyroid
storm through an identical mechanism. Preventive
administration of steroids for several months (2-3)
followed by a brief taper prior to initiating radioiodine
therapy is recommended for patients with mild,
moderate, or progressive ophthalmopathy. Patients
without obvious ophthalmopathy initially are at a much
lower risk of exacerbation with the start of radioactive
iodine treatment.
 Incorrect Answers:
 Answer 1: A larger dose of radioiodine could have led
to an even greater exacerbation of the patient's
exophthalmos. A smaller dose could have mitigated
the effect of the radioiodine.
 Answer 2: Beta-blockers act to control the symptoms
of hyperthyroidism, notably tachycardia and
palpitations; initiating a beta-blocker would not be
expected to decrease the risk or severity of the
worsening of this patient's ophthalmopathy.
 Answer 3: Antithyroid drugs should be stopped prior
to initiating therapy with radioactive iodine.
 Answer 5: Pretreatment with glucocorticoids has
been shown to decrease the risk or extent of
worsening exophthalmos after initiating radioactive
iodine therapy.
 A 33-year-old man is found to have lymphocytic
infiltration of his extraocular muscles secondary
to an autoimmune process. Fibroblasts in the
orbits of his eyes are dysregulated and large
amounts of glycosaminoglycans are deposited.
Which of the following is most likely present in
this patient? Topic Review Topic
 1. Infection of the orbit
 2. Decreased reabsorption of aqueous humor
 3. Autoimmune destruction of the lacrimal
glands
 4. Anti-TSH receptor antibodies
 5. Inflammation of the axial skeleton
 PREFERRED RESPONSE ▼ 4
 DISCUSSION: This patient's presentation is consistent with Graves' disease.
Graves' disease is an autoimmune disease characterized by stimulating anti-
TSH receptor antibodies.
 Graves' disease is the most common cause of hyperthyroidism. It presents
with non-tender thyroid enlargement, exophthalmos, pretibial myxedema,
and symptoms of hyperthyroidism. Exophthalmos occurs by autoimmune
attack on the extracellular muscles by lymphocytes, and subsequent
proliferation of fibroblasts with deposition of hyaluronic acid and
glycosaminoglycans. Treatment is with beta-blockers, thionamides, or I-131
ablation.
 Patel et al. discuss other autoimmune diseases with ocular manifestations
including RA, JRA, Sjogrens, SLE, MS, GCA, and the seronegative
spondyloarthropathies, whose "ocular symptoms may include dry or red
eyes, foreign-body sensation, pruritus, photophobia, pain, visual changes,
and even complete loss of vision."
 Bahn discusses the pathophysiology of Graves' disease ophthalmopathy in
NEJM: "orbital fibroblasts secrete large quantities of hyaluronan in response
to various cytokines, and a subgroup of orbital fibroblasts can differentiate
into mature adipocytes that have increased expression of thyrotropin
receptor. These cellular changes lead to the characteristically enlarged eye
muscles and expansion of orbital fat of patients with Graves'
ophthalmopathy."
 Incorrect Answers:
 Answer 1: Orbital cellulitis is an example of an
infection of the orbit. The question stem describes an
autoimmune process, not an infectious one.
Furthermore, exophthalmos is not usually seen in
orbital cellulitis.
 Answer 2: Decreased reabsorption of aqueous humor
through the canal of Schlemm describes glaucoma.
Glaucoma presents with insidious onset of peripheral
vision loss (open-angle glaucoma) or acute onset of
blurry vision (angle-closure glaucoma).
 Answer 3: Autoimmune destruction of the lacrimal and
salivary glands describes Sjogren's syndrome. Sjogren's
syndrome presents with dry eyes and dry mouth.
 Answer 5: Inflammation of the axial skeleton is one of
the hallmarks of ankylosing spondylitis. Anklyosing
spondylitis presents with insidious onset morning
stiffness in the back.
 A 53-year-old female presented to her primary
care physician for an annual check-up. She does
not smoke, use drugs, or consume alcohol. The
patient currently takes no medications. Routine
blood tests demonstrate a TSH level of 9.5 uU/mL
(normal 0.35 to 5.0 uU/ml). Her T3/T4 levels are
within normal limits. The initiation of treatment
for hypothyroidism would be indicated for all
EXCEPT which of the following findings?
1. Pretibial myxedema
2. Presence of antithyroid peroxidase (anti-TPO)
antibodies
3. Hyperlipidemia
4. Constipation
5. Decreased sweating
 PREFERRED RESPONSE ▼ 1
 DISCUSSION: This patient’s elevated TSH level coupled with
normal T3/T4 levels indicates subclinical hypothyroidism.
While patients with serum TSH levels greater than 10 uU/mL
are generally treated with levothyroxine, asymptomatic
patients with elevated TSH levels (but < 10) are not treated
with thyroid supplementation. Pretibial myxedema is a
symptom of clinically advanced HYPERthyroidism and does
not warrant thyroid supplementation.
 Subclinical hypothyroidism is found in approximately 2% of
the population, and may be found in as many as 20% of
women over age 60. Indications for treatment of
hypothyroidism with TSH levels below 10 uU/mL include the
presence of anti-TPO antibodies, clinical symptoms of
hypothyroidism, hyperlipidemia, and menstrual dysfunction.
Subclinical hypothyroidism is otherwise not treated due to
the risk of overtreatment with levothyroxine, which may
cause atrial fibrillation and bone mineral loss.
 A 40-year-old woman presents to her primary care
physician for an annual check-up. She complains of
intermittent fatigue. Laboratory tests are notable
for the following: TSH, serum: 11.2 uU/mL (normal
0.5-5.0 uU/mL) Thyroxine (T4), serum: 9.8 ug/dL
(normal 5-12 ug/dL) Which of the following other
conditions, if present, would merit treatment with
thyroxine? Topic Review Topic
 1. Tender thyroid gland
 2. High erythrocyte sedimentation rate (ESR)
 3. Hypercholesterolemia
 4. Proptosis
 5. Hand tremor
 PREFERRED RESPONSE ▼ 3
 CORRECT
 DISCUSSION: The patient presents with elevated TSH
(normal range 0.5-5.0 uU/mL) and T4 levels within
the normal reference range (5-12 ug/dL) consistent
with subclinical hypothyroidism. Patients with
subclinical hypothyroidism should be treated with
thyroxine if hypercholesterolemia is present.
 Subclinical hypothyroidism occurs in the early stages
of thyroid function inadequacy. The hypothalamic-
pituitary axis recognizes falling serum T4 levels and
increases TSH function accordingly. Patients with the
disease should be treated with thyroxine in the event
of goiter, hypercholesterolemia, symptoms of
hypothyroidism, or TSH levels > 20 uU/mL.
 Incorrect Answers:
 Answers 1 and 2: Painful, tender thyroid
gland and high ESR are associated with
hypothyroidism from subacute thyroiditis.
Subacute thyroiditis is managed with NSAIDs
and aspirin for mild symptoms and
corticosteroids for severe pain.
 Answer 4: Proptosis is a hallmark of Graves'
disease.
 Answer 5: Hand tremor is a symptom of
hyperthyroidism
A 37-year-old female presents to a general
medical clinic with muscle weakness. Review of
systems also reveals fatigue and chronic
constipation. Vital signs reveal HR 64, BP 110/80,
RR 12 and T 36.4. Physical examination is notable
for muscle weakness at the hips and shoulders.
Initial laboratory testing reveals a normal
erythrocyte sedimentation rate but an elevated
creatine kinase. What is the next step in
management?
1. Refer to a rheumatologist
2. Send thyroid stimulating hormone and T4
3. Send rheumatoid factor
4. Send ANA
5. Send AM Cortisol
 PREFERRED RESPONSE ▼ 2
 This patient has symptoms suggestive of hypothyroidism and associated
myopathy. Evaluation of TSH and free T4 are the first step in evaluating
for clinical hypothyroidism.
 Myopathy can be caused by hypothyroidism. Because it is not an
inflammatory-mediated myopathy, ESR should be normal, but CK will be
elevated. Typically, other symptoms of hypothyroidism are present, such
as fatigue, weight gain, and constipation. Other causes of myopathy
include infectious myopathies, inflammatory myopathies such as
dermatomyositis, inclusion body myopathies, and polymyositis, and drug
induced myopathies (especially from statins).
 Gaitonde et al. review hypothyroidism. Untreated hypothyroidism can
contribute to hypertension, dyslipidemia, infertility, cognitive
impairment, and neuromuscular dysfunction. The prevalence increases
with age, and is higher in females than in males. Autoimmune thyroid
disease is the most common etiology of hypothyroidism in the United
States. The best laboratory assessment of thyroid function is a serum
thyroid-stimulating hormone test.
 Kaminsky et al. review hypothyroid myopathy. Hypothyroidism induces a
metabolic myopathy, with a fall in energy production, especially
mitochondrial metabolism. This is due to a global inhibition of the main
oxidative pathways (substrate incorporation, substrate oxidation) and of
the respiratory chain.
 Incorrect Answers:
 Answer 1: Specialty referral in the setting of
hypothyroidism may include referral to an
endocrinologist, but this is not appropriate at this
time.
 Answer 3: Rheumatoid Factor is a screening test for
rheumatoid arthritis, but this patient's signs and
symptoms are more indicative of hypothyroidism.
 Answer 4: ANA is a high sensitivity, low specificity
test for Lupus.
 Answer 5: Morning cortisol would be appropriate if
you suspected Cushing's Disease, but the history is
more suggestive of hypothyroidism, which is also
much more common among the general population.
 A 31-year-old female with a past medical history of follicular
thyroid cancer treated two years prior with a total
thyroidectomy followed by radioactive iodine ablation
presents to her gynecologist with questions relating to
pregnancy. She and her partner are thinking about getting
pregnant, but she is concerned about her thyroid
replacement hormone, specifically the complications of being
hypothyroid during pregnancy. She wants to know how her
levothyroxine dosing should be handled prior to conception
and during her pregnancy. Which of the following would be an
appropriate response to this patient's concerns? Topic Review
Topic
 1. Your current dose will need to be increased before
conception to reduce the potential fetal complications.
 2. Your current dose will only need to be increased during
pregnancy, not before.
 3. Your current dose will only need to be increased before
pregnancy, not during.
 4. Your current dose will be sufficient for her and the fetus
during pregnancy.
 5. Your pregnancy is at increased risk of complications
because of hypothyroidism regardless of the dose.
 PREFERRED RESPONSE ▼ 2
 DISCUSSION: In hypothyroidism during pregnancy, the
levothyroxine dose will only need to be increased
during pregnancy, not before.
 The levothyroxine dose needs to be increased during
pregnancy to maintain a euthyroid state as a result of
the increase of thyroglobulin binding hormone in
pregnancy which decreases the amount of circulating
free T3/T4. Thyroid disease, if untreated during
pregnancy, increases the risk of miscarriage, placental
abruption, hypertensive disorders, and growth
restriction. While this patient was well-informed, it is
important for obstetricians to screen women at high
risk, including those with a history of thyroid disease,
type 1 diabetes mellitus, other autoimmune diseases,
current or past use of thyroid therapy, or a family
history of autoimmune thyroid disease.
 You are working up a 6-week-old American infant
in the pediatric clinic. He was born at term
following an uncomplicated pregnancy and now
lives with his parents in Massachusetts. However,
his chart reveals an indirect hyperbilirubinemia
present at birth. His mother reports poor feeding.
On physical exam you note lethargy, hypotonia, a
large, protruding tongue and coarse facial
features. Which of the following is the most likely
cause of this presentation? Topic Review Topic
 1. Iodine deficiency
 2. Thyroid dysgenesis
 3. Rett syndrome
 4. Trisomy 21
 5. Toxoplasmosis
 PREFERRED RESPONSE ▼ 2
 DISCUSSION: The child in this vignette most likely has
cretinism caused by untreated congenital
hypothyroidism as a result of thyroid dysgenesis, the
most common cause of cretinism in the developed
world.
 Cretinism is a condition that develops in children who
lack sufficient amounts of thyroid hormone. The classic
presentation is a child who appears normal at birth (as
there is maternal thyroid hormone present) but
gradually develops lethargy, hypotonia, coarse facial
features, and poor feeding over the next 6-12 weeks.
Cretinism can also be caused by iodine deficiency, the
most common cause in the developing world.
Congenital hypothyroidism can be caused by agenesis of
thyroid tissue or defects in the enzymes responsible for
thyroid hormone production. T4 is crucial during the
first two years of life for normal brain and bodily
development.
 Incorrect Answers
 Answer 1: While, iodine deficiency is the most
common cause of cretinism in the developing
world, it is less common than congenital
hypothyroidism due to thyroid agensis in the US.
 Answer 3: Rett syndrome has normal development
until 6-18 months with drooling, seizures, and
decreased head circumference.
 Answer 4: Down syndrome (trisomy 21), while a risk
factor for Hashimoto throiditis, presents with
macroglossia though would also have a flat facial
profile prominent epicentral folds, and simian
creases in hands.
 Answer 5: Congenital toxoplasmosis presents with
jaundice and hepatosplenomegaly but not the
other signs.
 These are incredibly common, and are palpable in as
much as 5% of women and 1% of men. Ninety-five
percent are benign (adenoma, colloid nodule, cyst).
 Thyroid nodules are rarely associated with clinically
apparent hyperfunctioning or hypofunctioning.
 Diagnostic Tests
 Thyroid nodules >1 em must be biopsied with a fine-
needle aspirate if there is normal thyroid function
(T4/TSH). Nodules in those who are euthyroid should
be biopsied. There is no need to ultrasound or do
radionuclide scanning because these tests cannot
exclude cancer.
 When a patient has a nodule:
1. Perform thyroid function tests (TSH and T4).
2. If tests are normal, biopsy the gland.
A 46-year-old woman comes to the office because
of a small mass she found on
palpation of her own thyroid. A small nodule is
found in the thyroid. There is no
tenderness. She is otherwise asymptomatic and
uses no medications.
What is the most appropriate next step in the
management of this patient?
a. Fine-needle aspiration
b. Radionuclide iodine uptake scan
c. T4 and TSH levels
d. Thyroid ultrasound
e. Surgical removal (excisional biopsy)
Answer: C. If the patient has a
hyperfunctioning gland (i.e., the T4 is elevated
or the TSH is decreased ), the patient does not
need immediate biopsy. Malignancy is not
hyperfunctioning. Ultrasound of thyroid is done
to evaluate the size of the lesion, but
does not change the need for either thyroid
function testing or needle aspiration.
Thyroid carcinoma
1. Workup of thyroid nodules
a. Thyroid nodules are usually benign and increase in
frequency with age.
b. Nodules should be evaluated with TSH levels, thyroid
function tests, ultrasound
(US), and fine needle aspiration (FNA) with biopsy.
c. “Cold” nodules exhibit decreased radioactive iodide
(I2) uptake (from decreased
metabolic activity); “hot” nodules exhibit increased
iodide uptake (from increased
metabolic activity).
d. Increased risk of malignancy 5 male, children, adults
over age 60 years and under
age 30 years, history of neck irradiation, poor iodide
uptake on thyroid scan
(cold nodule), solid nodule on US
e. Malignant nodules can arise from a variety of thyroid
cell types (see Table 5-7).
2. H/P
nontender nodule in anterior neck, dysphagia,
hoarseness; possible cervical
lymphadenopathy
3. Labs
biopsy provides diagnosis; thyroid hormones
normal or decreased
4. Radiology
US used to determine size and local extension;
thyroid scan may differentiate
hot from cold nodule (malignant nodules more
likely to be cold)
5. Treatment
a. Benign small cystic nodules may be observed.
b. Benign solid nodules are treated with surgery,
radioablation, and postoperative levothyroxine to
stop thyroid hormone overproduction and decrease
risk of malignant conversion.
c. Malignant tumors require surgical resection
(lobectomy for nonanaplastic tumors,1 cm
diameter, total thyroidectomy for larger tumors)
and radioiodine ablation.
d. Radiation therapy for tumors with local
extension; chemotherapy for metastatic tumors
e. Thyroid replacement (levothyroxine) needed
after surgery
Endocrinology thyroid disorder

More Related Content

What's hot

What's hot (20)

Disorders of the thyroid gland
Disorders of the thyroid glandDisorders of the thyroid gland
Disorders of the thyroid gland
 
Hypopituitarism
HypopituitarismHypopituitarism
Hypopituitarism
 
Hypothyroidism: Evaluation & Management by Dr Selim
Hypothyroidism: Evaluation & Management by Dr SelimHypothyroidism: Evaluation & Management by Dr Selim
Hypothyroidism: Evaluation & Management by Dr Selim
 
Hyperthyroidism
HyperthyroidismHyperthyroidism
Hyperthyroidism
 
Thyroid disorders
Thyroid disorders Thyroid disorders
Thyroid disorders
 
Diseases of thyroid gland
Diseases of thyroid glandDiseases of thyroid gland
Diseases of thyroid gland
 
Hypothyroidism final draft
Hypothyroidism final draftHypothyroidism final draft
Hypothyroidism final draft
 
Hyperthyriodism and graves disease
Hyperthyriodism and graves diseaseHyperthyriodism and graves disease
Hyperthyriodism and graves disease
 
Management of Thyroid Diseases & Emergencies
Management of Thyroid Diseases & EmergenciesManagement of Thyroid Diseases & Emergencies
Management of Thyroid Diseases & Emergencies
 
Parathyroid disorders
Parathyroid disorders Parathyroid disorders
Parathyroid disorders
 
Thyroid
ThyroidThyroid
Thyroid
 
Thyroid disorders
Thyroid disordersThyroid disorders
Thyroid disorders
 
Hypothyroidism.
Hypothyroidism.Hypothyroidism.
Hypothyroidism.
 
Goiter
GoiterGoiter
Goiter
 
Hypothyroidism
HypothyroidismHypothyroidism
Hypothyroidism
 
Thyroid Gland and Disease of Thyroid Gland
Thyroid Gland and Disease of Thyroid GlandThyroid Gland and Disease of Thyroid Gland
Thyroid Gland and Disease of Thyroid Gland
 
Thyroid disorders
Thyroid disordersThyroid disorders
Thyroid disorders
 
Hypothyroidism and Hyperthyroidism
Hypothyroidism and HyperthyroidismHypothyroidism and Hyperthyroidism
Hypothyroidism and Hyperthyroidism
 
Hypopituitarism
HypopituitarismHypopituitarism
Hypopituitarism
 
Hypothyroidism
HypothyroidismHypothyroidism
Hypothyroidism
 

Viewers also liked

Thyroid disorders treatment
Thyroid disorders treatmentThyroid disorders treatment
Thyroid disorders treatmentDr Sujay Patil
 
Clinical Diagnosis of Thyroid Disorders
Clinical Diagnosis of Thyroid DisordersClinical Diagnosis of Thyroid Disorders
Clinical Diagnosis of Thyroid DisordersReynaldo Joson
 
HYPERTHYROIDISM PART-1 BY DR BASHIR ASSOCIATE PROFESSOR MEDICINE SOPORE KASHMIR
HYPERTHYROIDISM PART-1 BY DR BASHIR ASSOCIATE PROFESSOR MEDICINE SOPORE KASHMIRHYPERTHYROIDISM PART-1 BY DR BASHIR ASSOCIATE PROFESSOR MEDICINE SOPORE KASHMIR
HYPERTHYROIDISM PART-1 BY DR BASHIR ASSOCIATE PROFESSOR MEDICINE SOPORE KASHMIRProf Dr Bashir Ahmed Dar
 
Thyroid disorders Part 1
Thyroid disorders Part 1Thyroid disorders Part 1
Thyroid disorders Part 1Pratap Tiwari
 
Thyroid disorders
Thyroid disordersThyroid disorders
Thyroid disordersHrudi Sahoo
 
Diseases of thyroid gland
Diseases of thyroid glandDiseases of thyroid gland
Diseases of thyroid glandraj kumar
 
Endocrinology Lect2,3 2003
Endocrinology Lect2,3  2003Endocrinology Lect2,3  2003
Endocrinology Lect2,3 2003Miami Dade
 
basic introductionary lecture on endocrinology and thyroid
basic introductionary lecture on endocrinology and thyroidbasic introductionary lecture on endocrinology and thyroid
basic introductionary lecture on endocrinology and thyroidPranesh Pawaskar
 
Hypo hyper thyroid graves
Hypo hyper thyroid gravesHypo hyper thyroid graves
Hypo hyper thyroid gravesmandira dahal
 
Endocrinology -pituitary gland
Endocrinology -pituitary glandEndocrinology -pituitary gland
Endocrinology -pituitary glandLih Yin Chong
 
Hyperthyroidism management
Hyperthyroidism managementHyperthyroidism management
Hyperthyroidism managementhodmedicine
 
Hyperthyroidism
HyperthyroidismHyperthyroidism
Hyperthyroidismfitango
 
Patho2 chapter40 student1
Patho2 chapter40 student1Patho2 chapter40 student1
Patho2 chapter40 student1btruong1
 

Viewers also liked (20)

Thyroid Health
Thyroid HealthThyroid Health
Thyroid Health
 
Thyroid disorders treatment
Thyroid disorders treatmentThyroid disorders treatment
Thyroid disorders treatment
 
Clinical pharmacy (thyroid disorder)
Clinical pharmacy (thyroid disorder)Clinical pharmacy (thyroid disorder)
Clinical pharmacy (thyroid disorder)
 
Clinical Diagnosis of Thyroid Disorders
Clinical Diagnosis of Thyroid DisordersClinical Diagnosis of Thyroid Disorders
Clinical Diagnosis of Thyroid Disorders
 
HYPERTHYROIDISM PART-1 BY DR BASHIR ASSOCIATE PROFESSOR MEDICINE SOPORE KASHMIR
HYPERTHYROIDISM PART-1 BY DR BASHIR ASSOCIATE PROFESSOR MEDICINE SOPORE KASHMIRHYPERTHYROIDISM PART-1 BY DR BASHIR ASSOCIATE PROFESSOR MEDICINE SOPORE KASHMIR
HYPERTHYROIDISM PART-1 BY DR BASHIR ASSOCIATE PROFESSOR MEDICINE SOPORE KASHMIR
 
Thyroid disorders Part 1
Thyroid disorders Part 1Thyroid disorders Part 1
Thyroid disorders Part 1
 
Thyroid disorders
Thyroid disordersThyroid disorders
Thyroid disorders
 
Diseases of thyroid gland
Diseases of thyroid glandDiseases of thyroid gland
Diseases of thyroid gland
 
Survey of the B-Vitamins
Survey of the B-VitaminsSurvey of the B-Vitamins
Survey of the B-Vitamins
 
thyroid disorder
 thyroid disorder  thyroid disorder
thyroid disorder
 
Endocrinology Lect2,3 2003
Endocrinology Lect2,3  2003Endocrinology Lect2,3  2003
Endocrinology Lect2,3 2003
 
Metabolism
MetabolismMetabolism
Metabolism
 
basic introductionary lecture on endocrinology and thyroid
basic introductionary lecture on endocrinology and thyroidbasic introductionary lecture on endocrinology and thyroid
basic introductionary lecture on endocrinology and thyroid
 
Hypo hyper thyroid graves
Hypo hyper thyroid gravesHypo hyper thyroid graves
Hypo hyper thyroid graves
 
Hyperthyroidism
HyperthyroidismHyperthyroidism
Hyperthyroidism
 
Endocrinology -pituitary gland
Endocrinology -pituitary glandEndocrinology -pituitary gland
Endocrinology -pituitary gland
 
Hyperthyroidism management
Hyperthyroidism managementHyperthyroidism management
Hyperthyroidism management
 
Hyperthyroidism
HyperthyroidismHyperthyroidism
Hyperthyroidism
 
Patho2 chapter40 student1
Patho2 chapter40 student1Patho2 chapter40 student1
Patho2 chapter40 student1
 
Thyroid
ThyroidThyroid
Thyroid
 

Similar to Endocrinology thyroid disorder

Thyroid disease, hypo & hyper thyrodisim
Thyroid disease, hypo & hyper thyrodisimThyroid disease, hypo & hyper thyrodisim
Thyroid disease, hypo & hyper thyrodisimSara Fahad
 
Diagnosis and treatment of hypothyroidism.pptx
Diagnosis and treatment of hypothyroidism.pptxDiagnosis and treatment of hypothyroidism.pptx
Diagnosis and treatment of hypothyroidism.pptxvivianOkoli1
 
Anesthesia: Thyroid and Parathyroid
Anesthesia: Thyroid and ParathyroidAnesthesia: Thyroid and Parathyroid
Anesthesia: Thyroid and ParathyroidBashar Mudallal
 
Interpretation of laboratory thyroid function tests
Interpretation of laboratory thyroid function tests Interpretation of laboratory thyroid function tests
Interpretation of laboratory thyroid function tests Hussam Elmouzi
 
Thyroid Overview
Thyroid OverviewThyroid Overview
Thyroid OverviewMiami Dade
 
2727_Management of Thyroid Disorders.ppt
2727_Management of Thyroid Disorders.ppt2727_Management of Thyroid Disorders.ppt
2727_Management of Thyroid Disorders.pptibrahimosman57
 
Anesthesia consideration in thyroid surgeries By Dr Sardar Saud Abbas
Anesthesia consideration in thyroid surgeries By Dr Sardar Saud AbbasAnesthesia consideration in thyroid surgeries By Dr Sardar Saud Abbas
Anesthesia consideration in thyroid surgeries By Dr Sardar Saud AbbasKhyber Teaching hospital
 
Thyroid Disease2
Thyroid Disease2Thyroid Disease2
Thyroid Disease2Deep Deep
 
Case Study Evaluation .docx
              Case Study Evaluation                       .docx              Case Study Evaluation                       .docx
Case Study Evaluation .docxhallettfaustina
 
Thyroid gland disorders hyper and hypo0110017 (2)
Thyroid gland disorders hyper and hypo0110017 (2)Thyroid gland disorders hyper and hypo0110017 (2)
Thyroid gland disorders hyper and hypo0110017 (2)AbdelNourBawadekji
 
Anesthesia for thyroid_disease....day_2[1]
Anesthesia for thyroid_disease....day_2[1]Anesthesia for thyroid_disease....day_2[1]
Anesthesia for thyroid_disease....day_2[1]LalemAnteneh
 

Similar to Endocrinology thyroid disorder (20)

Thyrotoxicosis
ThyrotoxicosisThyrotoxicosis
Thyrotoxicosis
 
THROID CRISIS.pptx
THROID CRISIS.pptxTHROID CRISIS.pptx
THROID CRISIS.pptx
 
Thyroid disease, hypo & hyper thyrodisim
Thyroid disease, hypo & hyper thyrodisimThyroid disease, hypo & hyper thyrodisim
Thyroid disease, hypo & hyper thyrodisim
 
Diagnosis and treatment of hypothyroidism.pptx
Diagnosis and treatment of hypothyroidism.pptxDiagnosis and treatment of hypothyroidism.pptx
Diagnosis and treatment of hypothyroidism.pptx
 
Anesthesia: Thyroid and Parathyroid
Anesthesia: Thyroid and ParathyroidAnesthesia: Thyroid and Parathyroid
Anesthesia: Thyroid and Parathyroid
 
Interpretation of laboratory thyroid function tests
Interpretation of laboratory thyroid function tests Interpretation of laboratory thyroid function tests
Interpretation of laboratory thyroid function tests
 
Endocrinology
EndocrinologyEndocrinology
Endocrinology
 
Thyroid Overview
Thyroid OverviewThyroid Overview
Thyroid Overview
 
2727_Management of Thyroid Disorders.ppt
2727_Management of Thyroid Disorders.ppt2727_Management of Thyroid Disorders.ppt
2727_Management of Thyroid Disorders.ppt
 
Anesthesia consideration in thyroid surgeries By Dr Sardar Saud Abbas
Anesthesia consideration in thyroid surgeries By Dr Sardar Saud AbbasAnesthesia consideration in thyroid surgeries By Dr Sardar Saud Abbas
Anesthesia consideration in thyroid surgeries By Dr Sardar Saud Abbas
 
Thyroidstorm ppt
Thyroidstorm pptThyroidstorm ppt
Thyroidstorm ppt
 
Hypothyroidism
HypothyroidismHypothyroidism
Hypothyroidism
 
Hyperthyroidism_case_scenerio.pptx
Hyperthyroidism_case_scenerio.pptxHyperthyroidism_case_scenerio.pptx
Hyperthyroidism_case_scenerio.pptx
 
Thyroid Disease2
Thyroid Disease2Thyroid Disease2
Thyroid Disease2
 
Case Study Evaluation .docx
              Case Study Evaluation                       .docx              Case Study Evaluation                       .docx
Case Study Evaluation .docx
 
Hyperthyroidism.pptx
Hyperthyroidism.pptxHyperthyroidism.pptx
Hyperthyroidism.pptx
 
Hyperthyroidism approach to management- dr selim
Hyperthyroidism approach to management- dr selimHyperthyroidism approach to management- dr selim
Hyperthyroidism approach to management- dr selim
 
Benign thyroid diseases
Benign thyroid diseasesBenign thyroid diseases
Benign thyroid diseases
 
Thyroid gland disorders hyper and hypo0110017 (2)
Thyroid gland disorders hyper and hypo0110017 (2)Thyroid gland disorders hyper and hypo0110017 (2)
Thyroid gland disorders hyper and hypo0110017 (2)
 
Anesthesia for thyroid_disease....day_2[1]
Anesthesia for thyroid_disease....day_2[1]Anesthesia for thyroid_disease....day_2[1]
Anesthesia for thyroid_disease....day_2[1]
 

More from Lih Yin Chong

Management of abdominal trauma
Management of abdominal traumaManagement of abdominal trauma
Management of abdominal traumaLih Yin Chong
 
Peripheral Nerve block(ankle block,wrist block, digital block)
Peripheral Nerve block(ankle block,wrist block, digital block)Peripheral Nerve block(ankle block,wrist block, digital block)
Peripheral Nerve block(ankle block,wrist block, digital block)Lih Yin Chong
 
peptic ulcer disease
peptic ulcer diseasepeptic ulcer disease
peptic ulcer diseaseLih Yin Chong
 
Skin condition associated with disease
Skin condition associated with diseaseSkin condition associated with disease
Skin condition associated with diseaseLih Yin Chong
 
Evaluating of fetal heart tracing
Evaluating of fetal heart tracingEvaluating of fetal heart tracing
Evaluating of fetal heart tracingLih Yin Chong
 
Congestive heart failure
Congestive heart failureCongestive heart failure
Congestive heart failureLih Yin Chong
 
Cardiac rhythm disorders
Cardiac rhythm disordersCardiac rhythm disorders
Cardiac rhythm disordersLih Yin Chong
 
Coronary artery disease
Coronary artery diseaseCoronary artery disease
Coronary artery diseaseLih Yin Chong
 
Acute coronary syndrome
Acute coronary syndromeAcute coronary syndrome
Acute coronary syndromeLih Yin Chong
 
Disease of adrenal gland
Disease of adrenal glandDisease of adrenal gland
Disease of adrenal glandLih Yin Chong
 
Endocrinology thyroid disorder
Endocrinology thyroid disorderEndocrinology thyroid disorder
Endocrinology thyroid disorderLih Yin Chong
 
Endocrinology parathyroid gland
Endocrinology parathyroid glandEndocrinology parathyroid gland
Endocrinology parathyroid glandLih Yin Chong
 

More from Lih Yin Chong (17)

Copd-2019
Copd-2019Copd-2019
Copd-2019
 
Management of abdominal trauma
Management of abdominal traumaManagement of abdominal trauma
Management of abdominal trauma
 
Peripheral Nerve block(ankle block,wrist block, digital block)
Peripheral Nerve block(ankle block,wrist block, digital block)Peripheral Nerve block(ankle block,wrist block, digital block)
Peripheral Nerve block(ankle block,wrist block, digital block)
 
Esophageal disease
Esophageal diseaseEsophageal disease
Esophageal disease
 
peptic ulcer disease
peptic ulcer diseasepeptic ulcer disease
peptic ulcer disease
 
Skin condition associated with disease
Skin condition associated with diseaseSkin condition associated with disease
Skin condition associated with disease
 
Orthopedics
OrthopedicsOrthopedics
Orthopedics
 
Breast pathology
Breast pathologyBreast pathology
Breast pathology
 
Evaluating of fetal heart tracing
Evaluating of fetal heart tracingEvaluating of fetal heart tracing
Evaluating of fetal heart tracing
 
Congestive heart failure
Congestive heart failureCongestive heart failure
Congestive heart failure
 
Cardiac rhythm disorders
Cardiac rhythm disordersCardiac rhythm disorders
Cardiac rhythm disorders
 
Coronary artery disease
Coronary artery diseaseCoronary artery disease
Coronary artery disease
 
Acute coronary syndrome
Acute coronary syndromeAcute coronary syndrome
Acute coronary syndrome
 
Diabetes mellitus
Diabetes mellitusDiabetes mellitus
Diabetes mellitus
 
Disease of adrenal gland
Disease of adrenal glandDisease of adrenal gland
Disease of adrenal gland
 
Endocrinology thyroid disorder
Endocrinology thyroid disorderEndocrinology thyroid disorder
Endocrinology thyroid disorder
 
Endocrinology parathyroid gland
Endocrinology parathyroid glandEndocrinology parathyroid gland
Endocrinology parathyroid gland
 

Recently uploaded

Romantic Opera MUSIC FOR GRADE NINE pptx
Romantic Opera MUSIC FOR GRADE NINE pptxRomantic Opera MUSIC FOR GRADE NINE pptx
Romantic Opera MUSIC FOR GRADE NINE pptxsqpmdrvczh
 
Earth Day Presentation wow hello nice great
Earth Day Presentation wow hello nice greatEarth Day Presentation wow hello nice great
Earth Day Presentation wow hello nice greatYousafMalik24
 
Quarter 4 Peace-education.pptx Catch Up Friday
Quarter 4 Peace-education.pptx Catch Up FridayQuarter 4 Peace-education.pptx Catch Up Friday
Quarter 4 Peace-education.pptx Catch Up FridayMakMakNepo
 
AmericanHighSchoolsprezentacijaoskolama.
AmericanHighSchoolsprezentacijaoskolama.AmericanHighSchoolsprezentacijaoskolama.
AmericanHighSchoolsprezentacijaoskolama.arsicmarija21
 
Crayon Activity Handout For the Crayon A
Crayon Activity Handout For the Crayon ACrayon Activity Handout For the Crayon A
Crayon Activity Handout For the Crayon AUnboundStockton
 
Computed Fields and api Depends in the Odoo 17
Computed Fields and api Depends in the Odoo 17Computed Fields and api Depends in the Odoo 17
Computed Fields and api Depends in the Odoo 17Celine George
 
EPANDING THE CONTENT OF AN OUTLINE using notes.pptx
EPANDING THE CONTENT OF AN OUTLINE using notes.pptxEPANDING THE CONTENT OF AN OUTLINE using notes.pptx
EPANDING THE CONTENT OF AN OUTLINE using notes.pptxRaymartEstabillo3
 
Employee wellbeing at the workplace.pptx
Employee wellbeing at the workplace.pptxEmployee wellbeing at the workplace.pptx
Employee wellbeing at the workplace.pptxNirmalaLoungPoorunde1
 
Alper Gobel In Media Res Media Component
Alper Gobel In Media Res Media ComponentAlper Gobel In Media Res Media Component
Alper Gobel In Media Res Media ComponentInMediaRes1
 
AMERICAN LANGUAGE HUB_Level2_Student'sBook_Answerkey.pdf
AMERICAN LANGUAGE HUB_Level2_Student'sBook_Answerkey.pdfAMERICAN LANGUAGE HUB_Level2_Student'sBook_Answerkey.pdf
AMERICAN LANGUAGE HUB_Level2_Student'sBook_Answerkey.pdfphamnguyenenglishnb
 
ENGLISH6-Q4-W3.pptxqurter our high choom
ENGLISH6-Q4-W3.pptxqurter our high choomENGLISH6-Q4-W3.pptxqurter our high choom
ENGLISH6-Q4-W3.pptxqurter our high choomnelietumpap1
 
Hierarchy of management that covers different levels of management
Hierarchy of management that covers different levels of managementHierarchy of management that covers different levels of management
Hierarchy of management that covers different levels of managementmkooblal
 
Procuring digital preservation CAN be quick and painless with our new dynamic...
Procuring digital preservation CAN be quick and painless with our new dynamic...Procuring digital preservation CAN be quick and painless with our new dynamic...
Procuring digital preservation CAN be quick and painless with our new dynamic...Jisc
 
What is Model Inheritance in Odoo 17 ERP
What is Model Inheritance in Odoo 17 ERPWhat is Model Inheritance in Odoo 17 ERP
What is Model Inheritance in Odoo 17 ERPCeline George
 
Types of Journalistic Writing Grade 8.pptx
Types of Journalistic Writing Grade 8.pptxTypes of Journalistic Writing Grade 8.pptx
Types of Journalistic Writing Grade 8.pptxEyham Joco
 
Introduction to ArtificiaI Intelligence in Higher Education
Introduction to ArtificiaI Intelligence in Higher EducationIntroduction to ArtificiaI Intelligence in Higher Education
Introduction to ArtificiaI Intelligence in Higher Educationpboyjonauth
 

Recently uploaded (20)

OS-operating systems- ch04 (Threads) ...
OS-operating systems- ch04 (Threads) ...OS-operating systems- ch04 (Threads) ...
OS-operating systems- ch04 (Threads) ...
 
9953330565 Low Rate Call Girls In Rohini Delhi NCR
9953330565 Low Rate Call Girls In Rohini  Delhi NCR9953330565 Low Rate Call Girls In Rohini  Delhi NCR
9953330565 Low Rate Call Girls In Rohini Delhi NCR
 
Romantic Opera MUSIC FOR GRADE NINE pptx
Romantic Opera MUSIC FOR GRADE NINE pptxRomantic Opera MUSIC FOR GRADE NINE pptx
Romantic Opera MUSIC FOR GRADE NINE pptx
 
Earth Day Presentation wow hello nice great
Earth Day Presentation wow hello nice greatEarth Day Presentation wow hello nice great
Earth Day Presentation wow hello nice great
 
Quarter 4 Peace-education.pptx Catch Up Friday
Quarter 4 Peace-education.pptx Catch Up FridayQuarter 4 Peace-education.pptx Catch Up Friday
Quarter 4 Peace-education.pptx Catch Up Friday
 
Model Call Girl in Bikash Puri Delhi reach out to us at 🔝9953056974🔝
Model Call Girl in Bikash Puri  Delhi reach out to us at 🔝9953056974🔝Model Call Girl in Bikash Puri  Delhi reach out to us at 🔝9953056974🔝
Model Call Girl in Bikash Puri Delhi reach out to us at 🔝9953056974🔝
 
AmericanHighSchoolsprezentacijaoskolama.
AmericanHighSchoolsprezentacijaoskolama.AmericanHighSchoolsprezentacijaoskolama.
AmericanHighSchoolsprezentacijaoskolama.
 
Crayon Activity Handout For the Crayon A
Crayon Activity Handout For the Crayon ACrayon Activity Handout For the Crayon A
Crayon Activity Handout For the Crayon A
 
Computed Fields and api Depends in the Odoo 17
Computed Fields and api Depends in the Odoo 17Computed Fields and api Depends in the Odoo 17
Computed Fields and api Depends in the Odoo 17
 
EPANDING THE CONTENT OF AN OUTLINE using notes.pptx
EPANDING THE CONTENT OF AN OUTLINE using notes.pptxEPANDING THE CONTENT OF AN OUTLINE using notes.pptx
EPANDING THE CONTENT OF AN OUTLINE using notes.pptx
 
Employee wellbeing at the workplace.pptx
Employee wellbeing at the workplace.pptxEmployee wellbeing at the workplace.pptx
Employee wellbeing at the workplace.pptx
 
Alper Gobel In Media Res Media Component
Alper Gobel In Media Res Media ComponentAlper Gobel In Media Res Media Component
Alper Gobel In Media Res Media Component
 
AMERICAN LANGUAGE HUB_Level2_Student'sBook_Answerkey.pdf
AMERICAN LANGUAGE HUB_Level2_Student'sBook_Answerkey.pdfAMERICAN LANGUAGE HUB_Level2_Student'sBook_Answerkey.pdf
AMERICAN LANGUAGE HUB_Level2_Student'sBook_Answerkey.pdf
 
ENGLISH6-Q4-W3.pptxqurter our high choom
ENGLISH6-Q4-W3.pptxqurter our high choomENGLISH6-Q4-W3.pptxqurter our high choom
ENGLISH6-Q4-W3.pptxqurter our high choom
 
Hierarchy of management that covers different levels of management
Hierarchy of management that covers different levels of managementHierarchy of management that covers different levels of management
Hierarchy of management that covers different levels of management
 
Model Call Girl in Tilak Nagar Delhi reach out to us at 🔝9953056974🔝
Model Call Girl in Tilak Nagar Delhi reach out to us at 🔝9953056974🔝Model Call Girl in Tilak Nagar Delhi reach out to us at 🔝9953056974🔝
Model Call Girl in Tilak Nagar Delhi reach out to us at 🔝9953056974🔝
 
Procuring digital preservation CAN be quick and painless with our new dynamic...
Procuring digital preservation CAN be quick and painless with our new dynamic...Procuring digital preservation CAN be quick and painless with our new dynamic...
Procuring digital preservation CAN be quick and painless with our new dynamic...
 
What is Model Inheritance in Odoo 17 ERP
What is Model Inheritance in Odoo 17 ERPWhat is Model Inheritance in Odoo 17 ERP
What is Model Inheritance in Odoo 17 ERP
 
Types of Journalistic Writing Grade 8.pptx
Types of Journalistic Writing Grade 8.pptxTypes of Journalistic Writing Grade 8.pptx
Types of Journalistic Writing Grade 8.pptx
 
Introduction to ArtificiaI Intelligence in Higher Education
Introduction to ArtificiaI Intelligence in Higher EducationIntroduction to ArtificiaI Intelligence in Higher Education
Introduction to ArtificiaI Intelligence in Higher Education
 

Endocrinology thyroid disorder

  • 1.
  • 2.  Main secretory hormone T4; T3 more potent  Free hormone only active 1.FT4 2.FT3(#2)  TSH is the best initial test for diagnosis(#1)  TBG (#3)( increase=endogenous T4)  Antimicrosomal antibody  Radioactive iodine uptake scan(RAIS)-check functional status of thyroid
  • 3.  Total T3,T4 are not always accurate: 1. Increase TBG level: - Pregnancy - OCP 2. Decrease TBG level: -nephrotic syndrome -liver ds
  • 4.
  • 5.  An 86-year-old female is brought to the emergency department from her nursing home facility. She has been more lethargic and less communicative over the past 3 days. The patient's only complaint is generalized weakness. Her past medical history is significant for mild dementia, well-controlled hypertension, and hyperlipidemia. Her medications include hydrochlorothiazide, losartan, and simvastatin. Triage vitals are T 99.0F, HR 104; RR 20; BP 120/86mmHg. EKG is shown in Figure A. Blood counts and metabolic panel are within normal limits. In addition to an infection work-up, which of the following would be the most useful to include in the evaluation of this patient?  1. Creatinine kinase  2. Calcium, magnesium, phosphorus levels  3. Liver function tests  4. Thyroid stimulating hormone  5. Lead level
  • 6.
  • 7.  PREFERRED RESPONSE ▼ 4  In an elderly patient presenting with new-onset atrial fibrillation (EKG in Figure A) and hypoactive altered mental status, suspect apathetic thyrotoxicosis, an atypical presentation of hyperthyroidism. A thyroid simulating hormone level (TSH) is the most appropriate screening test for hyperthyroidism.  The diagnosis of hyperthyroidism in the elderly patient requires a high index of suspicion, as the disease does not always present with the classic symptoms of hyperthyroidism, such as heat intolerance, weight loss, and hyperactivity. In elderly patients, the most common presenting symptoms are cardiac, specifically atrial fibrillation, congestive heart failure, and angina. Furthermore, these symptoms can be confused with primary cardiac pathology or masked by home medications (i.e. beta-blockers). Apathetic thyrotoxicosis manifests with apathy, weakness, depression, and lethargy.  In their review, Espino et al. discuss the work-up of altered mental status in the elderly patient. They recommend first distinguishing delirium from dementia or depression based on the patient's history and exam. A Mini-Mental State Exam may be helpful in determining the degree of impairment (Illustration A). Treatment is based on the underlying cause.  In a case series of three patients presenting with apathetic thyrotoxicosis, Arnold et al. report presenting symptoms of anorexia, gradual weight loss, restlessness, lethargy, weakness, dyspnea on exertion, depression, irritability, forgetfulness, anhedonia, CHF, dry skin, pedal edema, and polymyalgia. The three patients in this case series were female age 59-62 years old.  Figure A shows an EKG of a patient in atrial fibrillation.  Illustration A shows a Mini-Mental State Examination. The test is comprised of 30 points assessing for attention, memory, language, orientation, and registration. A score below 24 is considered abnormal.
  • 8.  Incorrect Answers:  Answer 1: A creatinine kinase level may be useful in a patient who was found down for a prolonged or undetermined amount of time.  Answer 2: Although hypocalcemia can result in altered mental status, in a patient with new onset atrial fibrillation without complaint of abdominal pain, tetany, or muscle cramps, hyperthyroidism should be ruled out first.  Answer 3: Liver function tests (LFTs) should be obtained in a patient with known or suspected liver disease where hepatic encephalopathy is suspected. Hepatic encephalopathy presents with new-onset dementia, seizures, coma, and asterixis.  Answer 5: A lead level should be obtained in a patient presenting with anemia, abdominal colic, and encephalopathy.
  • 9.
  • 10. 1 2 3
  • 11.
  • 12.  Graves ds(MC)  Toxic multinodular goiter  Subacute thyroiditis  Extrathyroid source -thyrotoxicosis factitious -stroma ovarii  Excess TSH (rare)
  • 13.
  • 14.
  • 15.
  • 16.  Hyperthyroidism + diffuse goiter + exophthalmus + Dermatopathy  Women(4th decade)  Autoantibody (Thyroid stimulating Ig)  Ass with other Autoimmune disorders: -pernicious anemia -myasthenia -diabetes
  • 17.  Most common cause of hyperthyroidism  An autoimmune disease with stimulating anti-TSH receptor antibodies  a type II hypersensitivity  anti-microsomal, anti-thyroglobulin antibodies also present  Female dominant HLA-B8, Dr3 association  Often incited during stress e.g. childbirth, infection, steroid withdrawal Physical exam  symmetrical, non-tender thyroid enlargement  ophthalmopathy (proptosis, exophthalmos) due to glycosaminoglycan deposition  pretibial myxedema  digital swelling Serology  ↑ total serum T4  ↑ free T4  ↓ serum TSH  diffusely ↑ 123I uptake
  • 18. Treatment Medical  β-blockers  thionamides  result in reduced hormone synthesis  PTU and methimazole  131I ablation  hypothyroidism may result  May cause transient worsening of exophtalmos or hyperthyroid symptoms due to release of thyroid hormone with thyroid cell destruction  Prevention: pretreatment with glucocorticoids Prognosis, Prevention, and Comlications  Stress-induced catecholamine surge may be fatal by arrhythmia  Pregnancy complications anti-TSH receptor antibodies may cross placenta and produce hyperthryoidism in the fetus
  • 19.
  • 20.
  • 21.  Nervous system(yonger)  Inability to sleep,tremor  Diarrhea,sweat  Heat intolerance  Weight loss(despite increase appetite)  Dypnea,palpitation,angina,cardiac failure,myopathy(old pt)  Pretibial myxedema  Palmar erythema  Menstrual irregularity  Osteoporosis,hypercalcemia
  • 22.
  • 23.
  • 24.  Thyroid studies(#1) -suppress TSH -serum FT3,4 elevated  RAI uptake(diffuse uptake) dy/dx solitary uptake=toxic nodular goiter  Thyroid stimulating Ig (TSI) (usually not needed)
  • 25. Treatment of Acute Hyperthyroidism and "Thyroid Storm" 1. Propranolol: blocks target organ effect, inhibits peripheral conversion of T4T3 2. Thiourea drugs (methimazole and propylthiouracil) : blocks hormone production(PTU safer in pregnancy) 3. Iodinated contrast material (iopanoic acid and ipodate): blocks the peripheral conversion of T4 to the more active T3; also blocks the release of existing hormone 4. Steroids (hydrocortisone) 5. Radioactive iodine: ablates the gland for a permanent cure
  • 26.  Thiourea drug may cause neutropenia(agranulocytosis)stop and change medication immediately  Long term treatment: -RAI ablationfollow them twice a year prevent them getting hypothyroidism -Thyroidectomy (rarely)-->reserve and use in pregnancy because we cant use RAI ablation in pregnancy  PregnancyMethimazole is contraindicated.
  • 27.  Steroids are the best initial therapy. Radiation is used in those not responding to steroids. Severe cases may need decompressive surgery
  • 28.  Non-autoimmune  Consequences of simple goiter  Nodular goiter on examination  Disease of the elderly  Presentation -Arrthymia, CHF(cardiac manifestation)  No opthalmopathy  Treatment: radioactive iodine
  • 29.
  • 30.  Extreme form of thyrotoxicosis  Precipitated by stress,surgery or trauma  Manifestation: -High fever + cardiac manifestation -hypotension,CHF -delirium,coma
  • 31.
  • 32.
  • 33.  Primary(95%) 1. Chronic thyroiditis(hashimato) 2. Post-ablative surgery,RAI 3. Drugs: lithium,amiodarone 4. Heritable biosynthethic defect(enzymatic deficiency)(rare) 5. Iodine deficiency  Suprathyroid cause: pituitary induces (secondary) or hypothalamic induced(tertiary)
  • 34.  Acromegaly  Bilateral carpal tunnel syndrome  Cold intolerance  Pseudodementia  Increase weight  Dry hair and skin  Hoarse voice  Slow deep tendion reflex  Elevated cholesterol
  • 35.
  • 36.  Cretinism(in newborn) and juvenile hypothyroidism.  Persistent physiology jaundice,hoarse cry,constipation,somnolence and feeding problem.  In later months,delayed mile-stones and dwarfism, coarse features, protruding tongue, broad flat nose, widely set eyes, sparse hair, dry skin ,protuberant abdomen, potbelly with umbilical hernia, impaired mental development, retarded bone age, and delayed dentition.
  • 37.
  • 38.
  • 39.  Restore metabolic state  Levothyroxine (T4) administered with monitoring of TSH/T3 level (it takes 6 weeks to stabilize after dosing changes).  If there is a strong suspicion of suprethyroid hypothyroidism/hypothalamic /pituitary origin given hydrocortisone first then replace thyroid hormone
  • 40.
  • 41.
  • 42.
  • 43.
  • 44. Older age presenting as:  Mental confusion  Hypothermia  Bradycardia  Hyponatremia  Hypoglycemia  Hypercapnia  Leukocytopenia  Decrease hematocrit  increaseCPK  Decrease EKG voltage
  • 45.
  • 46.
  • 47.
  • 48.
  • 49. Define:Inflammation of thyroid  Subacute thyroiditis  Hashimato thyroiditis  Lymphocytic thyroiditis  Reidel thyroiditis
  • 50.  Granulomatous,giant cell,or the Quervan thyroiditis  Viral origin and follows upper respiratory infection symptoms include: -malaise,fever,pain over thyroid,and pain refer to lower jaw,ears,neck and arms.  Thyroid gland is enlarged and firm  Lab test: 1.Increase ESR 2.Initial increase T3 and T4 follow by hyopthyroidism 3. decrease RAI uptake  Rx: symptomatic NSAIDs,prednisone and propanolol.  May smolder for months but eventually wil lreturn to normal function
  • 51.  A 34-year-old woman presents with a pain in the neck when with a mass. These symptoms started following a viral upper respiratory infection she had last week. On exam her thyroid is enlarged and tender to palpation. A biopsy is shown. (subacute granulomatous thyroiditis)
  • 52.
  • 53.
  • 54.
  • 55.
  • 56.
  • 57.
  • 58.
  • 59. 27year old woman comes to your office for “throat pain” for 1week and palpitation. She had no prior hx . Not taking any medication. BP 142/92,PP:112. Oropharynx normal ; thyroid painful and diffusely enlarged. TSH < 0.01 T4 : 14.4 (increase) RAI uptake scan 3%(low) What would be the next most appropriate test? Treatment?
  • 60.  Subacute Thyroiditis.  Check ESR  Treatment : only symptomatic treatment (pain)
  • 61. 32yo man admitted to ICU with delirium is treated with presumed sepsis. Thyroid hormone drawn on the day of admission show a very low FT3. The TSH is normal. There is concern about myxedema coma given that he had 2 history of “thyroid nodule”. What is the most likely diagnosis.
  • 62.  Apparent alterations thyroid hormone level with normal thyroid gland funciton  peripheral activity of outer ring deiodinase is ↓ normally converts T4 to T3  peripheral activity of inner ring deiodinase is ↑ normally converts T4 to reverse T3  Associated with  cancer  cardiac disease (MI, CHF)  CRF  sepsis Serology  ↓ total serum T4/T3  ↓ serum TSH  ↑ reverse T3  Management  Treat and manage primary underlying illness  Likely no need to start thyroid hormone replacement (controversial)  Recheck thyroid function studies after illness has resolved.
  • 63.  A 73-year-old female with a history of COPD and hypertension presents to the emergency room with a primary complaint of altered mental status. Her husband reports that the patient was suffering from worsening fatigue and a productive cough over the last several days. Chest radiographs in the emergency department suggest a diagnosis of pneumonia. Blood cultures are obtained and ultimately grow S. pneumoniae. The patient is admitted to the ICU for management of pneumonia and sepsis. During the course of her admission, thyroid function studies are ordered and show decreased levels of total serum T3 and T4, high-normal TSH, and increased levels of reverse T3. Which of the following is the most appropriate next step in management of this patient's abnormal thyroid function test results?
  • 64. A. Initiate thyroxine B. Radioactive iodine thyroid uptake test C. Thyroid ultrasound D. Fine needle aspiration thyroid biopsy E. Continue to monitor patient and recheck thyroid function after illness resolves
  • 65.  This patient with pneumonia and sepsis as well as decreased levels of thyroid hormones is suffering from euthyroid sick syndrome. Management of euthyroid sick syndrome entails treatment of the primary underlying disease and rechecking thyroid function after resolution of the illness.  Euthyroid sick syndrome is characterized by abnormal thyroid function tests concurrent with non-thyroid illness. Specifically, there are increased levels of reverse T3 and decreased levels of total T3. The effect on total T4 and TSH is more variable and less predictable. Euthyroid sick syndrome may present in the setting of underlying malignancy, cardiac disease such as myocardial infarction or congestive heart failure, renal failure, or sepsis.  Pittman reviews the evaluation and management of patients with abnormal thyroid function tests. Patients who are critically ill may have a wide array of abnormal thyroid function tests secondary to sick euthyroid syndrome. For the majority of patients, these abnormal results are due to the underlying disease and NOT a primary thyroid disorder. Management includes, first and foremost, addressing the underlying illness, with repeat thyroid function testing after recovery.  McIver and Gorman review euthyroid sick syndrome. The thyroid hormone abnormalities seen are likely due to cytokines or other inflammatory mediators acting on the hypothalamus, pituitary, and/or thyroid gland. The degree of thyroid hormone disturbance has been shown to correlate with the severity of illness; lower levels of thyroid hormones may predict a poorer prognosis. The use of thyroid hormone replacement therapy in euthyroid sick syndrome remains controversial.
  • 66.  Answer A: There are no prospective studies available that demonstrate the efficacy or utility of thyroid replacement therapy for euthyroid sick syndrome. Thyroid function abnormalities are typically reversible with treatment and resolution of the underlying causative illness.  Answers B,C: Imaging studies such as thyroid ultrasound or radioactive thyroid uptake test are not necessary in euthyroid sick syndrome.  Answer D: Thyroid biopsy is utilized to rule out thyroid cancer and diagnose thyroid nodules identified on imaging studies; it has no utility in euthyroid sick syndrome.
  • 67.  A 27-year-old female presents to the emergency room with palpitations. Vitals are stable with the exception of tachycardia. On physical examination, she appears gaunt but has no goiter or proptosis. Serum thyroglobulin and TSH are low. T3 is elevated. A radioactive iodine study is performed which shows low uptake in the thyroid gland. What is the most likely diagnosis? Topic Review Topic  1. Graves' disease  2. Toxic multinodular goiter  3. Factitious thyrotoxicosis  4. Papillary thyroid carcinoma  5. Iatrogenic hypothyroidism
  • 68.  PREFERRED RESPONSE ▼ 3  This patient has signs and symptoms of hyperthyroidism. Low uptake of radioactive iodine in conjunction with a low TSH and thyroglobulin are characteristic of excessive doses of levothyroxine (or factitious thyrotoxicosis).  Patients with factitious thyrotoxicosis have signs and symptoms of thyrotoxicosis without goiter or exophthalmos. Consider this diagnosis in a patient who has access to levothyroxine such as a nurse or a pharmacist. Recall the other common causes of hyperthyroidism: Graves' disease (diffuse toxic goiter), Plummer's disease (multinodular toxic goiter), toxic thyroid adenoma, Hashimoto's thyroiditis (which may cause transient hyperthyroidism before causing hypothyroidism), postpartum thyroiditis, and iodine induced hyperthyroidism. All these patients may present with the classic clinical features including nervousness, hand tremor, sweating, weight loss, diarrhea, and palpitations. Treatment may be pharmacologic via thionamides such as PTU and methimazole, or may consist of radioactive iodine 131, and even surgical subtotal thyroidectomy.
  • 69.  incorrect Answers:  Answer 1: In Graves' disease, the patient would have a goiter and the radioiodide scan would show diffuse uptake.  Answer 2: In toxic multinodular goiter (Plummer's disease), there would be patchy uptake on the thyroid scan. This form of hyperthyroidism is also more common in elderly patients.  Answer 4: Thyroid carcinomas are usually non- functioning nodules.  Answer 5: This patient's symptoms are indicative of hyperthyroidism, not hypothyroidism. Iatrogenic hypothyroidism occurs after radioiodine therapy, thyroidectomy, or may occur with certain medications such as Lithium.
  • 70.  A 32-year-old woman presents to your office with a one month history of heat intolerance, racing heart, unintentional 4 pound weight loss, and sweating. On physical examination there is a non-tender enlarged thyroid without evidence of proptosis or exophthalmos. Lab tests return with increased total T4, decreased TSH, and increased free T4. A radioactive iodine uptake exam is ordered and the imaging is displayed in Figure A. What is the most appropriate definitive treatment option for this patient? Topic Review Topic  FIGURES: A  1. Watchful waiting for symptom progression or recession  2. Beta-blockade alone  3. Anti-thyroid medications alone  4. Radioactive iodine ablation alone  5. Radioactive iodine ablation and prophylactic glucocorticoids
  • 71.  PREFERRED RESPONSE ▼ 4  DISCUSSION: The patient in the above vignette is experiencing symptoms associated with Graves' disease. In the US, the definitive treatment for Graves' with mild or no ophthalmopathy is radioactive iodine ablation.  Graves' disease is an autoimmune disease in which thyroid stimulating immunoglobulins bind to TSH-receptors. There tends to be female predominance, and symptoms are often precipitated by stress (e.g. childbirth, infection). Graves' often presents with heat intolerance, tachycardia, sweating, and a non-tender thyroid enlargement. Some characteristic physical exam findings are demonstrated in Illustrations A-C. Radioactive iodine uptake scans will show diffuse uptake. Indications for radioactive iodine over antithyroid agents include a large thyroid gland, multiple symptoms of thyrotoxicosis, high levels of thyroxine, and high titers of TSI. Additionally, anti-thyroid medication use alone has been shown to result in a higher rate of relapse when compared with radioactive iodine ablation.  Reid et al. explore hyperthyroidism causes and treatment options. Graves' disease is the most common cause of hyperthyroidism. Graves' can be treated with radioactive iodine, anti-thyroid medications, or surgery, but in the US, radioactive iodine ablation is the treatment of choice in patients without contraindications. Of note, an infiltrative ophthalmopathy is noted in approximately 50% of these patients.  Marinò et al. report on the genetic and non-genetic factors of Graves'. Some of the environmental factors that have been shown to be associated with this pathology are female gender, periods of immune reconstitution, immune modulation, iodine, smoking, and physiologic stress. The major genes that have been found to predispose patients to Graves' disease are: HLA complex, CD40, CTLA-4, PTPN22, FCRL3, Thyroglobulin, & TSH-R. With further study, the goal is to establish a causal treatment pattern based on the defining etiology and genetics involved.
  • 72.  Incorrect Answers:  Answer 1: Watchful waiting is not an appropriate treatment option for an individual with symptomatic Graves' disease.  Answer 2: Beta-blockade alone would help with some of the hyper-sympathetic symptoms of Graves' disease (e.g. tachycardia), but is not considered a definitive treatment.  Answer 3: In the US, anti-thyroid mediations alone would not be considered an appropriate treatment option for a patient with Graves' disease and associated symptomatic tachycardia that could be treated with beta-blockers as well. When methimazole or propylthiouracil is used alone, there is an increased risk of relapse of hyperthyroidism at a later time.  Answer 5: Radioactive iodine ablation with prophylactic glucocorticoids would be an appropriate treatment option for someone with moderate to severe exophthalmos as thyroid destruction during ablation can cause worsening of symptoms.
  • 73.  PREFERRED RESPONSE ▼ 4  DISCUSSION: The therapy for Graves' disease with the best long- term outcome is radioactive iodine therapy. Contraindications are pregnancy and severe ophthalmopathy.  Graves' disease (also known as diffuse toxic goiter) is the most common cause of hyperthyroidism, responsible for approximately 80% of all cases. It is an autoimmune disorder caused by the production of thyroid-stimulating antibody that bind to the TSH receptors on thyroid cells causing hormone synthesis. Treatment may include pharmacological therapy with propranolol for immediate control of symptoms as well as methimazole and propyltiouracil (PTU). However, the most successful therapy in the long term is radioactive iodine ablation therapy. This treatment is contraindicated in pregnant women in which PTU is preferred.  Recall the other common causes of hyperthyroidism: Plummer's disease, toxic thyroid adenoma, Hashimoto's thyroiditis, postpartum thyroiditis, iatrogenic hyperthyroidism, and iodine induced hyperthyoridism. All these patients may present with the classic clinical features including nervousness, hand tremor, sweating, weight loss, diarrhea, and palpitations. Treatment may be pharmacologic via thionamides such as PTU and methimazole, with radioactive iodine 131, or via surgical subtotal thyroidectomy.
  • 74.  Incorrect Answers:  Answer 1: Levothyroxine is the treatment for hypothyroidism but would make hyperthyroidism worse.  Answer 2: PTU is the therapeutic choice for a pregnant woman with Graves' but is inferior to radioactive iodine in the long term.  Answer 3: Methimazole is a pharmacologic option but it is not as effective as radioactive iodine in the long term. Notably, it is contraindicated in pregnancy.  Answer 5: Propranolol is good for immediate control of adrenergic symptoms but is not as efficacious as radioactive iodine in the long term.
  • 75.  A 45-year-old female presents to her primary care clinic with symptoms of palpitations, hyperhidrosis, tremor and general hyperactivity. A radioactive iodine update scan of this patient showing decreased thyroid uptake would effectively rule out which condition? Topic Review Topic  1. Subacute painless thyroiditis  2. Subacute granulomatous thyroiditis  3. Graves' disease  4. Iodine-induced thyroid toxicosis  5. Levothyroxineoverdose
  • 76.  REFERRED RESPONSE ▼ 3  DISCUSSION: The patient in this vignette most likely has hyperthyroidism, as demonstrated by her symptoms of hypermetabolism. It is important to distinguish high and low uptake forms of hyperthyroidism, of which the only high uptake-form listed is Graves disease.  There are several types of thyroiditis, one form of which is subacute granulomatous thyroiditis. This condition can present initially with either hypo or hyperthyroidism. In the case of initial hyperthyroidism, the destruction of the thyroid gland causes a release of previously synthesized thyroid hormone but will ultimately convert to a hypothyroid state as these stores are consumed but not replaced. It is important to note that the initial hyperthyroidism may progress to thyroid storm, which may be fatal. Subacute granulomatous thyroiditis can also initially present as a hypothyroid state usually following a viral illness, and in these cases is usually self-limited.
  • 77.
  • 78.  ) A 61-year-old man presents to the emergency room complaining a racing heart, sweats, and diarrhea for 2 weeks. Review of systems is positive for unintentional weight loss of 10 pounds in 1 month. Serum TSH is found to be 0.02 mIU/L (normal 0.5 - 5.0 mIU/L). The patient is shown in Figure A. If the patient is treated with I-131 radioiodine therapy, which of the following is the most likely complication?  FIGURES: A  1. Agranulocytosis  2. Increased total cancer mortality  3. Hyperthyroidism  4. Hypothyroidism  5. Hypoparathyroidism
  • 79.  PREFERRED RESPONSE ▼ 4  DISCUSSION: The clinical presentation is consistent with Graves' disease. Hypothyroidism (due to radiation thyroiditis) may occur in the treatment of Graves' disease with I-131 radioiodine therapy.  In Graves' disease, the entire thyroid gland is hyperfunctional. Uptake of the radioactive isotope of iodine throughout the gland results in effective ablation but may destroy too much thyroid tissue, resulting in a hypothyroid state. More than 75% of patients become hypothyroid following radioactive iodine thyroid ablation.
  • 80.  Incorrect Answers: Answer 1: Agranulocytosis is the most feared complication of the anti-thyroid drugs propylthiouracil and methimazole. Answer 2: Ron et al., in a study of over 35,000 patients, found no increased risk of total cancer mortality following I-131 treatment. There may be a slightly elevated risk of thyroid cancer mortality following I-131 treatment, but underlying thyroid disease appears to play a role. Answer 3: While possible if insufficient dose of I-131 is given, hypothyroidism is much more common. Answer 5: Hypoparathyroidism occurs if the parathyroid glands are incidentally removed during total thyroidectomy.
  • 81.  A 69-year-old man presents to the general medical clinic with palpitations. He has a history of an endocrine disorder that he reports has caused him to have chronic diarrhea and weight loss. His vital signs are temperature 37 degrees Celsius, blood pressure 130/85, heart rate 141, and respiratory rate of 18 with an oxygen saturation of 99% on room air. His pulse is irregular on physical examination. He is mentating normally and is in no acute distress. His exam is also notable for hyperreflexia and enlargement around his neck. An EKG reveals the following in figure A. What would be the next best step in management of this patient's chief complaint? Topic Review Topic  FIGURES: A  1. Emergent cardioversion  2. Administration of propranolol  3. Administration of amiodarone  4. Iodine 131 ablation  5. Administration of methimazole
  • 82.
  • 83.  PREFERRED RESPONSE ▼ 2  DISCUSSION: In a patient with atrial fibrillation or tachycardia secondary to hyperthyroidism, the initial appropriate treatment is a beta blocker if that patient is stable.  Atrial fibrillation occurs when conduction in the atria fires in a continuous chaotic pattern resulting in an irregular, rapid ventricular rate. There are multiple causes including coronary artery disease, myocardial infarction, hypertension, mitral valve disease, pericarditis, pulmonary disease, alcohol intake, and stress. Another cause is hyperthyroidism. Clinically patients present with palpitations, dizziness, angina, and an irregular pulse. On EKG, the clinician should look for irregular RR intervals and tachycardia. Treatment depends on whether the patient is stable or unstable. In the unstable patient, immediate electrical cardioversion to sinus rhythm is indicated. In a stable patient, treatment focuses on rate control (target 60- 100) with beta blockers or calcium channel blockers. After rate is controlled, rhythm can be addressed with cardioversion.
  • 84.  Image A depicts the classic irregularly irregular tachycardia indicative of atrial fibrillation. Note the absence of P waves.  Incorrect Answers:  Answer 1: Emergent cardioversion is indicated in management of atrial fibrillation when the patient is not stable. However, this patient is stable and could be treated with a beta blocker.  Answer 3: Administration of amiodarone is used in the treatment of post myocardial infarction arrhythmias such as ventricular tachycardia but would not be indicated in this case.  Answers 4 and 5: These are potential options in the management of Grave's hyperthyroidism but would not immediately address the patient's chief complaint which is palpitations secondary to atrial fibrillation and tachycardia.
  • 85.  A 36-year-old G1P0 female presents to labor and delivery in the 38th week of her pregnancy and undergoes an uncomplicated spontaneous vaginal delivery. Shortly after birth, the child is noted to have dysphagia, irritability, frequent stooling, and increased appetite. The mother notes no history of drug or medication use during the pregnancy. The newborn's CBC is within normal limits. Thyroid studies reveal an increased free T4 in the newborn. Which of the following is the most likely the cause of this infant's presentation?  1. Initial presentation of DiGeorge syndrome  2. Intrauterine toxoplasmosis infection  3. Maternal iodine deficiency  4. Maternal history of Graves' disease treated with radioactive thyroid ablation 10 years ago  5. Maternal history of Hashimoto's thyroiditis
  • 86.  PREFERRED RESPONSE ▼ 4  DISCUSSION: Pregnant mothers with Graves' disease, even after being treated surgically with thyroidectomy, can have persistent levels of thyroid stimulating immunoglobulin that can cross the placenta and cause thyrotoxicosis in the newborn, as seen in this vignette.  Graves' disease is the most common cause of hyperthyroidism. This condition is an autoimmune disorder with stimulating anti-TSH receptor antibodies (type II hypersensitivity). Anti-microsomal and anti- thyroglobulin antibodies are also present. Graves' disease is a female dominant disorder and has associations with HLA-B8 and DR3. Thyrotoxicosis is most often incited during stress (e.g. childbirth, infection, or steroid withdrawal). Fetal hyperthyroidism can present with low birth weight, microcephaly, warm wet skin, dysphagia, irritability, increased appetite with poor weight gain, exophthalmos, and diffuse goiter.  Bishnoi and Sachmechi discuss hyperthyroid disease management during pregnancy. Thyroid-stimulating hormone (TSH) is induced in normal pregnancy; however, in patients who are already hyperthyroid this may result in thyrotoxicosis leading to abortion, stillbirth, neonatal death, and low birth weight. They report that the main cause of thyrotoxicosis in pregnancy is Graves' disease, which may be treated with antithyroid drugs or surgery. Importantly, many thyroid conditions and treatments may directly affect the fetus, such as autoimmune postpartum thyroiditis.
  • 87.  Incorrect Answers:  Answer 1: Hypoparathyroidism, not hyperthyroidism, is seen in DiGeorge syndrome.  Answer 2: Intrauterine toxoplasmosis infection is not associated with hyperthyroidism.  Answer 3: Maternal iodine deficiency is associated with hypothyroidism, not hyperthyroidism. Congenital hypothyroidism presents with poor feeding, lethargy, hypotonia, coarse facial features, large protruding tongue, and developmental delay at 6-12 weeks of life.  Answer 5: Maternal history of Hashimoto's thyroiditis is not associated with neonatal hyperthyroidism.
  • 88.  A 32-year-old woman with Graves' disease is undergoing treatment with radioactive iodine. Her initial presentation consisted of symptoms of sweating, weight-loss, and intermittent palpitations along with a physical examination significant for mild-to-moderate exophthalmos. After completing one week of radioactive iodine therapy, she reports worsening of her proptosis, with increased pain and worsened periorbital edema. Which of the following could have prevented the worsening of this patient's exophthalmos? Topic Review Topic  1. Giving a larger dose of radioiodine therapy  2. Initiation of beta-blocker at time of radioiodine therapy  3. Begin methimazole concurrent with initiating radioiodine therapy  4. Pre-treatment with prednisone prior to initiating radioiodine therapy  5. This is an expected outcome from radioactive iodine therapy, no preventive options are availab
  • 89.
  • 90.  PREFERRED RESPONSE ▼ 4  DISCUSSION: This patient's Graves' ophthalmopathy was worsened by the initiation of radioactive iodine. Pretreatment with glucocorticoids, such as prednisone, may prevent this adverse effect.  Worsening of exophthalmos is due to the release of excess thyroid hormone during the destruction of thyroid cells by the radioactive iodine. Administration of radioactive iodine may also precipitate a thyroid storm through an identical mechanism. Preventive administration of steroids for several months (2-3) followed by a brief taper prior to initiating radioiodine therapy is recommended for patients with mild, moderate, or progressive ophthalmopathy. Patients without obvious ophthalmopathy initially are at a much lower risk of exacerbation with the start of radioactive iodine treatment.
  • 91.  Incorrect Answers:  Answer 1: A larger dose of radioiodine could have led to an even greater exacerbation of the patient's exophthalmos. A smaller dose could have mitigated the effect of the radioiodine.  Answer 2: Beta-blockers act to control the symptoms of hyperthyroidism, notably tachycardia and palpitations; initiating a beta-blocker would not be expected to decrease the risk or severity of the worsening of this patient's ophthalmopathy.  Answer 3: Antithyroid drugs should be stopped prior to initiating therapy with radioactive iodine.  Answer 5: Pretreatment with glucocorticoids has been shown to decrease the risk or extent of worsening exophthalmos after initiating radioactive iodine therapy.
  • 92.  A 33-year-old man is found to have lymphocytic infiltration of his extraocular muscles secondary to an autoimmune process. Fibroblasts in the orbits of his eyes are dysregulated and large amounts of glycosaminoglycans are deposited. Which of the following is most likely present in this patient? Topic Review Topic  1. Infection of the orbit  2. Decreased reabsorption of aqueous humor  3. Autoimmune destruction of the lacrimal glands  4. Anti-TSH receptor antibodies  5. Inflammation of the axial skeleton
  • 93.  PREFERRED RESPONSE ▼ 4  DISCUSSION: This patient's presentation is consistent with Graves' disease. Graves' disease is an autoimmune disease characterized by stimulating anti- TSH receptor antibodies.  Graves' disease is the most common cause of hyperthyroidism. It presents with non-tender thyroid enlargement, exophthalmos, pretibial myxedema, and symptoms of hyperthyroidism. Exophthalmos occurs by autoimmune attack on the extracellular muscles by lymphocytes, and subsequent proliferation of fibroblasts with deposition of hyaluronic acid and glycosaminoglycans. Treatment is with beta-blockers, thionamides, or I-131 ablation.  Patel et al. discuss other autoimmune diseases with ocular manifestations including RA, JRA, Sjogrens, SLE, MS, GCA, and the seronegative spondyloarthropathies, whose "ocular symptoms may include dry or red eyes, foreign-body sensation, pruritus, photophobia, pain, visual changes, and even complete loss of vision."  Bahn discusses the pathophysiology of Graves' disease ophthalmopathy in NEJM: "orbital fibroblasts secrete large quantities of hyaluronan in response to various cytokines, and a subgroup of orbital fibroblasts can differentiate into mature adipocytes that have increased expression of thyrotropin receptor. These cellular changes lead to the characteristically enlarged eye muscles and expansion of orbital fat of patients with Graves' ophthalmopathy."
  • 94.  Incorrect Answers:  Answer 1: Orbital cellulitis is an example of an infection of the orbit. The question stem describes an autoimmune process, not an infectious one. Furthermore, exophthalmos is not usually seen in orbital cellulitis.  Answer 2: Decreased reabsorption of aqueous humor through the canal of Schlemm describes glaucoma. Glaucoma presents with insidious onset of peripheral vision loss (open-angle glaucoma) or acute onset of blurry vision (angle-closure glaucoma).  Answer 3: Autoimmune destruction of the lacrimal and salivary glands describes Sjogren's syndrome. Sjogren's syndrome presents with dry eyes and dry mouth.  Answer 5: Inflammation of the axial skeleton is one of the hallmarks of ankylosing spondylitis. Anklyosing spondylitis presents with insidious onset morning stiffness in the back.
  • 95.  A 53-year-old female presented to her primary care physician for an annual check-up. She does not smoke, use drugs, or consume alcohol. The patient currently takes no medications. Routine blood tests demonstrate a TSH level of 9.5 uU/mL (normal 0.35 to 5.0 uU/ml). Her T3/T4 levels are within normal limits. The initiation of treatment for hypothyroidism would be indicated for all EXCEPT which of the following findings? 1. Pretibial myxedema 2. Presence of antithyroid peroxidase (anti-TPO) antibodies 3. Hyperlipidemia 4. Constipation 5. Decreased sweating
  • 96.  PREFERRED RESPONSE ▼ 1  DISCUSSION: This patient’s elevated TSH level coupled with normal T3/T4 levels indicates subclinical hypothyroidism. While patients with serum TSH levels greater than 10 uU/mL are generally treated with levothyroxine, asymptomatic patients with elevated TSH levels (but < 10) are not treated with thyroid supplementation. Pretibial myxedema is a symptom of clinically advanced HYPERthyroidism and does not warrant thyroid supplementation.  Subclinical hypothyroidism is found in approximately 2% of the population, and may be found in as many as 20% of women over age 60. Indications for treatment of hypothyroidism with TSH levels below 10 uU/mL include the presence of anti-TPO antibodies, clinical symptoms of hypothyroidism, hyperlipidemia, and menstrual dysfunction. Subclinical hypothyroidism is otherwise not treated due to the risk of overtreatment with levothyroxine, which may cause atrial fibrillation and bone mineral loss.
  • 97.  A 40-year-old woman presents to her primary care physician for an annual check-up. She complains of intermittent fatigue. Laboratory tests are notable for the following: TSH, serum: 11.2 uU/mL (normal 0.5-5.0 uU/mL) Thyroxine (T4), serum: 9.8 ug/dL (normal 5-12 ug/dL) Which of the following other conditions, if present, would merit treatment with thyroxine? Topic Review Topic  1. Tender thyroid gland  2. High erythrocyte sedimentation rate (ESR)  3. Hypercholesterolemia  4. Proptosis  5. Hand tremor
  • 98.  PREFERRED RESPONSE ▼ 3  CORRECT  DISCUSSION: The patient presents with elevated TSH (normal range 0.5-5.0 uU/mL) and T4 levels within the normal reference range (5-12 ug/dL) consistent with subclinical hypothyroidism. Patients with subclinical hypothyroidism should be treated with thyroxine if hypercholesterolemia is present.  Subclinical hypothyroidism occurs in the early stages of thyroid function inadequacy. The hypothalamic- pituitary axis recognizes falling serum T4 levels and increases TSH function accordingly. Patients with the disease should be treated with thyroxine in the event of goiter, hypercholesterolemia, symptoms of hypothyroidism, or TSH levels > 20 uU/mL.
  • 99.  Incorrect Answers:  Answers 1 and 2: Painful, tender thyroid gland and high ESR are associated with hypothyroidism from subacute thyroiditis. Subacute thyroiditis is managed with NSAIDs and aspirin for mild symptoms and corticosteroids for severe pain.  Answer 4: Proptosis is a hallmark of Graves' disease.  Answer 5: Hand tremor is a symptom of hyperthyroidism
  • 100. A 37-year-old female presents to a general medical clinic with muscle weakness. Review of systems also reveals fatigue and chronic constipation. Vital signs reveal HR 64, BP 110/80, RR 12 and T 36.4. Physical examination is notable for muscle weakness at the hips and shoulders. Initial laboratory testing reveals a normal erythrocyte sedimentation rate but an elevated creatine kinase. What is the next step in management? 1. Refer to a rheumatologist 2. Send thyroid stimulating hormone and T4 3. Send rheumatoid factor 4. Send ANA 5. Send AM Cortisol
  • 101.  PREFERRED RESPONSE ▼ 2  This patient has symptoms suggestive of hypothyroidism and associated myopathy. Evaluation of TSH and free T4 are the first step in evaluating for clinical hypothyroidism.  Myopathy can be caused by hypothyroidism. Because it is not an inflammatory-mediated myopathy, ESR should be normal, but CK will be elevated. Typically, other symptoms of hypothyroidism are present, such as fatigue, weight gain, and constipation. Other causes of myopathy include infectious myopathies, inflammatory myopathies such as dermatomyositis, inclusion body myopathies, and polymyositis, and drug induced myopathies (especially from statins).  Gaitonde et al. review hypothyroidism. Untreated hypothyroidism can contribute to hypertension, dyslipidemia, infertility, cognitive impairment, and neuromuscular dysfunction. The prevalence increases with age, and is higher in females than in males. Autoimmune thyroid disease is the most common etiology of hypothyroidism in the United States. The best laboratory assessment of thyroid function is a serum thyroid-stimulating hormone test.  Kaminsky et al. review hypothyroid myopathy. Hypothyroidism induces a metabolic myopathy, with a fall in energy production, especially mitochondrial metabolism. This is due to a global inhibition of the main oxidative pathways (substrate incorporation, substrate oxidation) and of the respiratory chain.
  • 102.  Incorrect Answers:  Answer 1: Specialty referral in the setting of hypothyroidism may include referral to an endocrinologist, but this is not appropriate at this time.  Answer 3: Rheumatoid Factor is a screening test for rheumatoid arthritis, but this patient's signs and symptoms are more indicative of hypothyroidism.  Answer 4: ANA is a high sensitivity, low specificity test for Lupus.  Answer 5: Morning cortisol would be appropriate if you suspected Cushing's Disease, but the history is more suggestive of hypothyroidism, which is also much more common among the general population.
  • 103.  A 31-year-old female with a past medical history of follicular thyroid cancer treated two years prior with a total thyroidectomy followed by radioactive iodine ablation presents to her gynecologist with questions relating to pregnancy. She and her partner are thinking about getting pregnant, but she is concerned about her thyroid replacement hormone, specifically the complications of being hypothyroid during pregnancy. She wants to know how her levothyroxine dosing should be handled prior to conception and during her pregnancy. Which of the following would be an appropriate response to this patient's concerns? Topic Review Topic  1. Your current dose will need to be increased before conception to reduce the potential fetal complications.  2. Your current dose will only need to be increased during pregnancy, not before.  3. Your current dose will only need to be increased before pregnancy, not during.  4. Your current dose will be sufficient for her and the fetus during pregnancy.  5. Your pregnancy is at increased risk of complications because of hypothyroidism regardless of the dose.
  • 104.  PREFERRED RESPONSE ▼ 2  DISCUSSION: In hypothyroidism during pregnancy, the levothyroxine dose will only need to be increased during pregnancy, not before.  The levothyroxine dose needs to be increased during pregnancy to maintain a euthyroid state as a result of the increase of thyroglobulin binding hormone in pregnancy which decreases the amount of circulating free T3/T4. Thyroid disease, if untreated during pregnancy, increases the risk of miscarriage, placental abruption, hypertensive disorders, and growth restriction. While this patient was well-informed, it is important for obstetricians to screen women at high risk, including those with a history of thyroid disease, type 1 diabetes mellitus, other autoimmune diseases, current or past use of thyroid therapy, or a family history of autoimmune thyroid disease.
  • 105.  You are working up a 6-week-old American infant in the pediatric clinic. He was born at term following an uncomplicated pregnancy and now lives with his parents in Massachusetts. However, his chart reveals an indirect hyperbilirubinemia present at birth. His mother reports poor feeding. On physical exam you note lethargy, hypotonia, a large, protruding tongue and coarse facial features. Which of the following is the most likely cause of this presentation? Topic Review Topic  1. Iodine deficiency  2. Thyroid dysgenesis  3. Rett syndrome  4. Trisomy 21  5. Toxoplasmosis
  • 106.  PREFERRED RESPONSE ▼ 2  DISCUSSION: The child in this vignette most likely has cretinism caused by untreated congenital hypothyroidism as a result of thyroid dysgenesis, the most common cause of cretinism in the developed world.  Cretinism is a condition that develops in children who lack sufficient amounts of thyroid hormone. The classic presentation is a child who appears normal at birth (as there is maternal thyroid hormone present) but gradually develops lethargy, hypotonia, coarse facial features, and poor feeding over the next 6-12 weeks. Cretinism can also be caused by iodine deficiency, the most common cause in the developing world. Congenital hypothyroidism can be caused by agenesis of thyroid tissue or defects in the enzymes responsible for thyroid hormone production. T4 is crucial during the first two years of life for normal brain and bodily development.
  • 107.  Incorrect Answers  Answer 1: While, iodine deficiency is the most common cause of cretinism in the developing world, it is less common than congenital hypothyroidism due to thyroid agensis in the US.  Answer 3: Rett syndrome has normal development until 6-18 months with drooling, seizures, and decreased head circumference.  Answer 4: Down syndrome (trisomy 21), while a risk factor for Hashimoto throiditis, presents with macroglossia though would also have a flat facial profile prominent epicentral folds, and simian creases in hands.  Answer 5: Congenital toxoplasmosis presents with jaundice and hepatosplenomegaly but not the other signs.
  • 108.  These are incredibly common, and are palpable in as much as 5% of women and 1% of men. Ninety-five percent are benign (adenoma, colloid nodule, cyst).  Thyroid nodules are rarely associated with clinically apparent hyperfunctioning or hypofunctioning.  Diagnostic Tests  Thyroid nodules >1 em must be biopsied with a fine- needle aspirate if there is normal thyroid function (T4/TSH). Nodules in those who are euthyroid should be biopsied. There is no need to ultrasound or do radionuclide scanning because these tests cannot exclude cancer.  When a patient has a nodule: 1. Perform thyroid function tests (TSH and T4). 2. If tests are normal, biopsy the gland.
  • 109. A 46-year-old woman comes to the office because of a small mass she found on palpation of her own thyroid. A small nodule is found in the thyroid. There is no tenderness. She is otherwise asymptomatic and uses no medications. What is the most appropriate next step in the management of this patient? a. Fine-needle aspiration b. Radionuclide iodine uptake scan c. T4 and TSH levels d. Thyroid ultrasound e. Surgical removal (excisional biopsy)
  • 110. Answer: C. If the patient has a hyperfunctioning gland (i.e., the T4 is elevated or the TSH is decreased ), the patient does not need immediate biopsy. Malignancy is not hyperfunctioning. Ultrasound of thyroid is done to evaluate the size of the lesion, but does not change the need for either thyroid function testing or needle aspiration.
  • 111. Thyroid carcinoma 1. Workup of thyroid nodules a. Thyroid nodules are usually benign and increase in frequency with age. b. Nodules should be evaluated with TSH levels, thyroid function tests, ultrasound (US), and fine needle aspiration (FNA) with biopsy. c. “Cold” nodules exhibit decreased radioactive iodide (I2) uptake (from decreased metabolic activity); “hot” nodules exhibit increased iodide uptake (from increased metabolic activity). d. Increased risk of malignancy 5 male, children, adults over age 60 years and under age 30 years, history of neck irradiation, poor iodide uptake on thyroid scan (cold nodule), solid nodule on US e. Malignant nodules can arise from a variety of thyroid cell types (see Table 5-7).
  • 112. 2. H/P nontender nodule in anterior neck, dysphagia, hoarseness; possible cervical lymphadenopathy 3. Labs biopsy provides diagnosis; thyroid hormones normal or decreased 4. Radiology US used to determine size and local extension; thyroid scan may differentiate hot from cold nodule (malignant nodules more likely to be cold)
  • 113. 5. Treatment a. Benign small cystic nodules may be observed. b. Benign solid nodules are treated with surgery, radioablation, and postoperative levothyroxine to stop thyroid hormone overproduction and decrease risk of malignant conversion. c. Malignant tumors require surgical resection (lobectomy for nonanaplastic tumors,1 cm diameter, total thyroidectomy for larger tumors) and radioiodine ablation. d. Radiation therapy for tumors with local extension; chemotherapy for metastatic tumors e. Thyroid replacement (levothyroxine) needed after surgery